LSPU Self-Paced Learning Module (SLM) : Calculus

Download as pdf or txt
Download as pdf or txt
You are on page 1of 88

Republic of the Philippines

Laguna State Polytechnic University


Province of Laguna
ISO 9001:2015 Certified
Level I Institutionally Accredited

LSPU Self-Paced Learning Module (SLM)


Course Calculus
Sem/AY First Semester/2020-2021
Module No. 1st
Lesson Title Relations and Function & Limits and Continuity of Algebraic Functions
Week
2nd Week – 4th Week
Duration
Date October 12 – 30, 2020
Description This lesson will discuss relations & functions, limits, and continuity. These topics are at
of the the heart of any first course in calculus. All the limits facts are stated and use graphs to
Lesson introduce the concept of the limit of the function. This lesson will also provide an
example in proving a limit using the formal definition of a limit.

Learning Outcomes
Intended Students should be able to meet the following intended learning outcomes:
Learning • Write detailed solutions using appropriate mathematical language
Outcomes • Identify areas in Mathematics and other fields where relations, functions and
limits of a functions is useful
• Generate solutions to unfamiliar problems and recognize the appropriate tools
of calculus to solve applied problems.

Targets/ At the end of the lesson, students should be able to:


Objectives • Define and explain the concept of relations and functions
• Describe and discuss the difference between relations and functions
• Sketch and interpret the graphs of relations and functions
• Classify and sketch the graphs of special types of functions
• Determine and describe the domain and range of relations and functions
• Use functional notations to solve real world problems
• Define and interpret the concept of limit intuitively
• Develop the skills in evaluating the limit of quotients one side limits, infinite
limits, and limits at infinity
• Apply the concept of limits and continuity to solve real world problems

LSPU SELF-PACED LEARNING MODULE: TECHNOLOGY FOR TEACHING AND LEARNING


Prepared by: John Louie S. Marasigan
Republic of the Philippines
Laguna State Polytechnic University
Province of Laguna
ISO 9001:2015 Certified
Level I Institutionally Accredited

Student Learning Strategies


Lecture Guide

Relation

A relation is a set of ordered pairs of mathematical quantities. The domain of a


relation is the set of first coordinates of the ordered pairs of that relation. The
range of relation is the set of second coordinates of the ordered pairs of that
relation.

The graph of a relation is the set of points in the plane that correspond to the
ordered pairs of that relation.

Inequality Symbols
< less than
> greater than
≤ less than or equal to
≥ grater than or equal to

Remarks:

Inequality is a statement that one mathematical quantity is less than (or greater
Offline Activities than) another. The solution set of inequality is the set of all numbers which when
(Self-Paced)Offline submitted for the variable, make the inequality a true statement.

Two types of Inequalities


• Conditional Inequality
• Unconditional Inequality

Conditional Inequality is an inequality which is true only for certain values of the
variable(s)

Unconditional Inequality is equality which is true for all values of the variables
or which contains no variables.

LSPU SELF-PACED LEARNING MODULE: TECHNOLOGY FOR TEACHING AND LEARNING


Prepared by: John Louie S. Marasigan
Republic of the Philippines
Laguna State Polytechnic University
Province of Laguna
ISO 9001:2015 Certified
Level I Institutionally Accredited

Interval Notation

(a,b) : Open Interval. All real numbers between, but not including a and b, a<b

[a,b] : Closed Interval . Closed Interval. All real numbers and including a and b, a
<b

(a,b],[a,b),(−∞,b],[a,∞) : Half-open or half-closed intervals. In each case the


interval contains one endpoint but not the other.

Remarks

An interval of real numbers is the set of all real numbers between two given
numbers (i.e. the endpoints of the interval) and one both or neither endpoint.

Example 1. Determine the domain and range of the set A defined by the relation

A = {(-2,-1), (-2,1),(-1,-2),(-1,2), (0,-3),(0,3),(1,-2),(1,2),(2,-1),(2,1)}

Solution: The domain of the set of all the first coordinates of the ordered pairs

Domain: {-2, -1,0,1,2}

While the range is the set of all the second coordinates of the ordered pairs.
Range: {-3, -2, -1,1,2,3}

Function

• If A and B are any two, then a function from A to B is a rule that assigns to
each element of A exactly one element of B
• If A and B are two sets, then a function from A to B is a set f of ordered
pairs in A x B with the property that for each a ∈ A there exists a unique b
∈ B with (a,b) ∈ f

A function f is a special type of a relation such that no two ordered pairs of the
set have different second coordinates. The set of all the first coordinates of the
ordered pairs is the domain of the function. The set of all second coordinates of
the ordered pairs is the range of the function

LSPU SELF-PACED LEARNING MODULE: TECHNOLOGY FOR TEACHING AND LEARNING


Prepared by: John Louie S. Marasigan
Republic of the Philippines
Laguna State Polytechnic University
Province of Laguna
ISO 9001:2015 Certified
Level I Institutionally Accredited
• In symbols, y = f(x) is read “y is a function of x”

Representation of Domain and Range


Source Note: Comandante, Felipe L. (2005) Differential Calculus made Easy (Metric Edition) National Bookstore

Remarks:

To be a function means no two ordered pairs of the set can have the same first
coordinate and different second coordinates. This means that no vertical line can
intersect the graph in more than one point.

Or, if we have a rule or formula giving y in terms x and there is no more than one
value of y for each value of x, then y is said to be a function of x.

Example: Sketch the graph and determine the domain and range of this function
y = f(x) = √𝑥 + 2

Solution: f(x) should be a real number, so that (x+2) must be greater than or
equal to 0

Graph of f(x) = √𝑥 + 2 using GeoGebra

Domain: {x | x ≥ −2 , x ∈ ℝ}
Range: {y | y≥ 0 , y ∈ ℝ}

LSPU SELF-PACED LEARNING MODULE: TECHNOLOGY FOR TEACHING AND LEARNING


Prepared by: John Louie S. Marasigan
Republic of the Philippines
Laguna State Polytechnic University
Province of Laguna
ISO 9001:2015 Certified
Level I Institutionally Accredited

The Linear Function

The simplest type of function is the linear function, where the graph is a
line. To graph a linear, we need only two points in keeping with Euclids Axiom:
“Two points determine a line” or “The shortest between two points is a line”.
Notice that when we say a line in mathematics, that means it is straight
otherwise, it is a curve.

Any equation of the form


y = mx + b

(where m is the slope of the line and b is the y-intercept) is called a linear
function in x.

Remarks

Notice that if m=0 , then f(x) = b , which we called a constant function. If the
domain of a constant function is the set of real numbers, then the graph of f(x) =
b is a horizontal line. The line parallel to the y-axis is called a vertical line. Notice
that vertical lines are not functions.

Example: Sketch the graph and determine the domain and range of

y = f(x) = 2x – 3 x ∈ [ -1, 3)

Solution: The domain of the linear function is restricted. Notice that when x=3,
the function is not defined and there is a hole in the graph, Thus x = -1 ,

y = f (-1) = 2(-1) -3 = -2-3 = -5


When assume that x =3 so that,

y = f(3) = 2(3) -3 = 6-3 = 3


Remember that x=3

Thus, the domain is the set of real numbers greater than or equal to -1 but less
than 3. Using set notation, we write

Domain : { x | -1 ≤ x ≤ 3, x ∈ ℝ}

The range is the set of real numbers greater than or equal to -5 but less than 3,
that is
Range : { y | -5 ≤ y ≤ 3, y ∈ ℝ}

LSPU SELF-PACED LEARNING MODULE: TECHNOLOGY FOR TEACHING AND LEARNING


Prepared by: John Louie S. Marasigan
Republic of the Philippines
Laguna State Polytechnic University
Province of Laguna
ISO 9001:2015 Certified
Level I Institutionally Accredited

Source Note: Comandante, Felipe L. (2005) Differential Calculus made Easy (Metric Edition) National Bookstore
Graph of a Linear Function y = f(x) = 2x – 3 x ∈ [ -1, 3)

Remarks
In future works with graphs, the solid dot on the graph indicates that the point
is part of the graph; an open dot indicates that the point is not part of the graph

The Quadratic Function

Any equation in the form of y =f(x) = ax2 + bx + c where a≠0 is called a quadratic
equation. The restriction that a≠0 is necessary because if a = 0, then the equation
becomes a linear function. The graph of a quadratic function is called a parabola.

In a quadratic function

y = f(x) = f(x) = ax2 + bx + c, a≠0

i. if a > 0, the parabola opens upward and has a minimum value at the vertex
ii. if a < 0, the parabola opens downward and has a maximum value at the vertex

The vertex of a parabola is given by the equation

𝑏 4𝑎𝑐 − 𝑏2 𝑏 4𝑎𝑐 − 𝑏2
x=- and y = ⇒ Vertex: - ,
2𝑎 4𝑎 2𝑎 4𝑎

LSPU SELF-PACED LEARNING MODULE: TECHNOLOGY FOR TEACHING AND LEARNING


Prepared by: John Louie S. Marasigan
Republic of the Philippines
Laguna State Polytechnic University
Province of Laguna
ISO 9001:2015 Certified
Level I Institutionally Accredited
Example: Sketch the graph and determine the vertex, domain, and range of

y =𝑥 2 and y =− 𝑥 2
Solution:
The vertices of these two parabolas lie at the origin. The first parabola opens
upward (since a = 1>0) while the second parabola opens downward (since a=-1
<0)

y =𝑥 2 y =−𝑥 2
Graphs of Quadratic Functions using GeoGebra

For y =𝑥 2 For y =−𝑥 2


Domain {x |x ∈ ℝ} {x |x ∈ ℝ}
Range {y |y ≥ 0,y ∈ ℝ} {y |y ≤ 0,y ∈ ℝ}
Vertex (0,0) (0,0)

The Cubic Function

Any equation in the form of

y = f(x) = f(x) = ax3 + bx2 + cx + d, a≠0

is called a cubic function in x

Example: Determine the domain and range and sketch the graph of

y =f(x) = x3 and y = f(x) = 1- x3

Solution: The graphs

For y = x3 For y =1 − x3
Domain {x |x ∈ ℝ} {x |x ∈ ℝ}
Range {y | y ∈ ℝ} {y | y ∈ ℝ}
Intercepts (0,0) (1,0) and (0,1)

LSPU SELF-PACED LEARNING MODULE: TECHNOLOGY FOR TEACHING AND LEARNING


Prepared by: John Louie S. Marasigan
Republic of the Philippines
Laguna State Polytechnic University
Province of Laguna
ISO 9001:2015 Certified
Level I Institutionally Accredited

y = x3 y =1 − x3
Graphs of Cubic Functions using GeoGebra

The Rational Function

Any function defined by


𝑝(𝑥)
𝑓(𝑥) =
, 𝑞(𝑥) ≠ 0
𝑞(𝑥)
where 𝑝(𝑥) and 𝑔(𝑥) are algebraic functions is called a rational function.

Remarks
Since any values of x such q(x) = 0 are excluded from the domain a rational
function usually has a graph with one or more breaks.
Example: Determine the domain and range and sketch the graphs of
1 1
y =f(x) = 𝑥 and y = f(x) = 𝑥 2

Solution: The graphs,


1 1
For y = For y =
𝑥 𝑥2
Domain {x |x ∈ ℝ, 𝑥 ≠ 0} {x |x ∈ ℝ, 𝑥 ≠ 0}
Range {y | y ∈ ℝ, 𝑦 ≠ 0} {y | y > 0, 𝑦 ∈ ℝ}
Intercepts (0,0) (1,0) and (0,1)

1 1
y= 𝑥 y = 𝑥2
Graphs of Rational Functions using GeoGebra

LSPU SELF-PACED LEARNING MODULE: TECHNOLOGY FOR TEACHING AND LEARNING


Prepared by: John Louie S. Marasigan
Republic of the Philippines
Laguna State Polytechnic University
Province of Laguna
ISO 9001:2015 Certified
Level I Institutionally Accredited

Special Types of Functions

The Absolute Value Function

The absolute value of a number is always positive and it can never be negative,
thus , the negative value function is defined by

x, x >0
f(x) = |x| = 0 x=0
-x, x<0

The graph is symmetric about the y-axis. The domain is the set of real numbers
while the range is the set of real numbers greater than or equal to zero . Hence

Domain : {x |x ∈ ℝ }
Range : {y |y ≥ 0 , y ∈ ℝ}

The Absolute Value using GeoGebra


The Signum Function

Signum is a latin word for sign. Except at zero, the value of the signum function
is determined by the algebraic sign of its argument (see Figure) When the
argument is positive, sgn x = +1 ; when x is negative , sgn x = -1. Thus,

1 x >0
f(x) = sgn x = 0 x=0
-1 x<0

The domain is the is the set of real numbers while the range are the integers -1,
0, and 1 . Hence,
Domain : {x |x ∈ ℝ }
Range : {-1 , 0, 1}

LSPU SELF-PACED LEARNING MODULE: TECHNOLOGY FOR TEACHING AND LEARNING


Prepared by: John Louie S. Marasigan
Republic of the Philippines
Laguna State Polytechnic University
Province of Laguna
ISO 9001:2015 Certified
Level I Institutionally Accredited

Source Note: Comandante, Felipe L. (2005) Differential Calculus made Easy (Metric Edition) National Bookstore
Signum Function

The Greatest Integer Function

The symbol [x] is defined as the greatest integer which is less than or equal to
the number x. The greatest integer function is defined by

f(x) = [x] = the largest integer n such that n ≤ x.

The domain is the set of real numbers while the range is the set of integers. Thus,

Domain: {x |x ∈ ℝ } and Range: {y |y ∈ ℤ }

To sketch the graph (see Figure), we consider the following intervals of length
one so that,
[x] = -3 if -3 ≤ x < -2
[x] = -2 if -2 ≤ x < -1
[x] = -1 if -1 ≤ x < 0
[x] = 0 if 0 ≤ x < 1
[x] = 1 if 1 ≤ x < 2
[x] = 2 if 2 ≤ x < 3 , and so on

Source Note: Comandante, Felipe L. (2005) Differential Calculus made Easy (Metric Edition) National Bookstore
The Greatest Integer Function

LSPU SELF-PACED LEARNING MODULE: TECHNOLOGY FOR TEACHING AND LEARNING


Prepared by: John Louie S. Marasigan
Republic of the Philippines
Laguna State Polytechnic University
Province of Laguna
ISO 9001:2015 Certified
Level I Institutionally Accredited

The Split Function

A function that is defined by different formulas for different parts of its domain
is called a

Example : Sketch the graph and determine the domain and range of

-x , x ≥0
f(x) =
1–x, x<0
Solution :

If x ≥ 0 then f(x) = -x . However, if x < 0 we use f(x) = 1-x to find the values of f
(see figure) . The domain is the set of real numbers while the range is the union
of the sets defined for y > 1 and y ≤ 0.

Domain: {x |x ∈ ℝ } and Range: {y |y > 1 , y ∈ ℝ } ∪ {y |y ≤ 0 , y ∈ ℝ }

Source Note: Comandante, Felipe L. (2005) Differential Calculus made Easy (Metric Edition) National Bookstore
Split Function Consisting of Two Functions

Exponential Functions

Lets start with b > 0, b ≠1. An exponential function is then a function in the form

f(x) = 𝑏 𝑥
Properties of f(x) = 𝑏 𝑥

1. f(0) = 1. The function will always take the value of 1 at x = 0


2. f(x) ≠ 0 . An exponential function will never be zero
3. f(x) > 0 . An exponential function is always positive

LSPU SELF-PACED LEARNING MODULE: TECHNOLOGY FOR TEACHING AND LEARNING


Prepared by: John Louie S. Marasigan
Republic of the Philippines
Laguna State Polytechnic University
Province of Laguna
ISO 9001:2015 Certified
Level I Institutionally Accredited
4. The previous two properties can be summarized by saving that the range of
an exponential function is (0, ∞ )
5. The domain of an exponential function is (-∞,∞ ). In other words, you can
plug every x into an exponential function
6. If 0 < b < 1 then
a. f(x) → 0 as x → ∞
b. f(x) → ∞ as x → − ∞
7. if b > 1 then,
a. f(x) → ∞ as x → ∞
b. f(x) → 0 as x → − ∞
Remarks:
The natural exponential function is f(x) = 𝑒 𝑥 , where e ≈2.71828
• Since e > 1 we also know that 𝑒 𝑥 → ∞ as x → ∞ and 𝑒 𝑥 →0 as x → -∞

Logarithm Functions

Lets start with b > 0, b ≠1. Then we have

y = logb x is equivalent to x = by
The first is logarithmic function form and the second is called the exponential
form. The number b is called the base

Special Logarithms
ln x = logex This log is called the natural logarithm
log x = log10x This log is called the common logarithm

Source Note: Paul’s Online Notes (2003-2020) Retrieved August 15, 2020, from https://tutorial.math.lamar.edu/

LSPU SELF-PACED LEARNING MODULE: TECHNOLOGY FOR TEACHING AND LEARNING


Prepared by: John Louie S. Marasigan
Republic of the Philippines
Laguna State Polytechnic University
Province of Laguna
ISO 9001:2015 Certified
Level I Institutionally Accredited

From this graph we can get a couple of nice properties about the natural
logarithm

ln 𝑥 → ∞ 𝑎𝑠 x → ∞
ln 𝑥 → − ∞ as x → 0 , x > 0

Properties
1. The Domain of the logarithm function is (0, ∞). In other words, we can only
plug positive numbers into a logarithm! We cannot plug in zero or a negative
number

2. The range of the logarithm function is (-∞ , ∞)

3. logbb = 1
4. logb1 = 0
5. logbbx = x
6. blogbx = x
Note:

f(x) = 𝑏 𝑥 and g(x) = logbx

are inverses of each other

7. logbxy = logbx + logby


𝑥
8. logb(𝑦) = logbx - logby

9. logb(𝑥 𝑟 ) = r logb x

The change base formula is,

𝑙𝑜𝑔 𝑥
logbx = 𝑙𝑜𝑔 𝑎b
𝑎

• Two most common change of base formula are

𝑙𝑛 𝑥 𝑙𝑜𝑔 𝑥
logbx = 𝑙𝑛 b and logbx = 𝑙𝑜𝑔 b

LSPU SELF-PACED LEARNING MODULE: TECHNOLOGY FOR TEACHING AND LEARNING


Prepared by: John Louie S. Marasigan
Republic of the Philippines
Laguna State Polytechnic University
Province of Laguna
ISO 9001:2015 Certified
Level I Institutionally Accredited
Inverse of a Function

Given the function f(x) we want to find the inverse function, 𝑓 −1 (x).

1. First replace f(x) with y. This is done to make the rest of the process easier.
2. Replace every x with a y and replace every y with an x
3. Solve the equation from Step 2 for y. This is the step where mistakes are
most often made so be careful with this step.
4. Replace y with 𝑓 −1 (x). In other words, we have managed to find the inverse
at this point.
5. Verify your work by checking that

(f ∘ 𝑓 −1 )(x) = x

and

(𝑓 −1 ∘ 𝑓 )(x) = x

are both true.

Example: Given f(x)=3x−2 find 𝑓 −1 (x)

f(x)=3x−2 ⇒ y = 3x−2
x = 3y -2
Solve for y
x + 2 = 3y
1
(x+2) =y
3
𝑥 2
+ =y
3 3

Finally replace y with 𝑓 −1 (x)

𝑥 2
𝑓 −1 (x) = +3
3
Checking:

(f ∘ 𝑓 −1 )(x) = f [𝑓 −1 (x)]
𝑥 2
=f +
3 3
𝑥 2
=3 + -2
3 3
= x + 2 -2
=x

LSPU SELF-PACED LEARNING MODULE: TECHNOLOGY FOR TEACHING AND LEARNING


Prepared by: John Louie S. Marasigan
Republic of the Philippines
Laguna State Polytechnic University
Province of Laguna
ISO 9001:2015 Certified
Level I Institutionally Accredited

Source Note: Paul’s Online Notes (2003-2020) Retrieved August 15, 2020, from https://tutorial.math.lamar.edu/

Limits

Tangent Line

The tangent line is defined as the line that intersects the curve at only one point
while the line that intersects the curve in two or more distinct points is called a
secant line.

The slope m of a line is defined as the tangent of its angle of inclination 𝜃 or


equivalently, as the ratio of the change in vertical distance (rise) to the change in
horizontal distance (run) as the point moves along the line in either direction.
Notice that the slope of a line is constant (See Figure A)

𝑟𝑖𝑠𝑒 △𝑦 𝑦 −𝑦
m = tan 𝜃 = = △𝑥 = 𝑥2 −𝑥1
𝑟𝑢𝑛 2 1

Source Note: Comandante, Felipe L. (2005) Differential Calculus made Easy (Metric Edition) National Bookstore
Figure A. Slope of a line

LSPU SELF-PACED LEARNING MODULE: TECHNOLOGY FOR TEACHING AND LEARNING


Prepared by: John Louie S. Marasigan
Republic of the Philippines
Laguna State Polytechnic University
Province of Laguna
ISO 9001:2015 Certified
Level I Institutionally Accredited

The slope of a curve is not constant and must be determined for each particular
point of interest .Let the points 𝑃1 (𝑥1 , 𝑦1 ) and 𝑃2 (𝑥2 , 𝑦2 ) be any two points on
the curve. So that we can draw a secant line between the two points on the curve
(See Figure B). The slope of the secant line joining the points 𝑃1 and 𝑃2 is given
by
△𝑦 𝑦 −𝑦
𝑚𝑠𝑒𝑐 = △𝑥 = 𝑥2 −𝑥1
2 1

Source Note: Comandante, Felipe L. (2005) Differential Calculus made Easy (Metric Edition) National Bookstore
Figure B. Slope of a Curve

Let the point 𝑃1 be fixed while point 𝑃2 be moved along the curve y = f(x)
towards the 𝑃1 . As the 𝑃2 is moved along the curve y = f(x) towards the point 𝑃1
The slope of the line joining the two points will in general vary. That is , as the
point 𝑃2 moves closer and closer to the point 𝑃1 , the slope of the secant line varies
by smaller and smaller amounts and approaches a constant limiting value. When
this occurs, the limiting value is said to be the slope of a curve at that point. That
is, the first derivative is interpreted as the slope of a curve at that point. Hence,

Δ𝑦 𝑦2 −𝑦1
𝑚𝑡𝑎𝑛 = lim 𝑚𝑠𝑒𝑐 = lim = lim
Δ𝑥 ⟶ 0 Δ𝑥 ⟶ 0 Δ𝑥 Δ𝑥 ⟶ 0 𝑥2 −𝑥1

𝑑𝑦
Let 𝑚𝑡𝑎𝑛 = f ‘(x) = 𝑑𝑥
where , Δ𝑥 = 𝑥2 - 𝑥1 ⟹ 𝑥2 = 𝑥1 + Δ𝑥
𝑦1 = f(𝑥1 ) and 𝑦2 = f(𝑥2 )

By substitution , we get

𝑑𝑦 Δ𝑦 𝑦2 −𝑦1 𝑓(𝑥2 )− 𝑓(𝑥1 ) 𝑓(𝑥1 + Δ𝑥)− 𝑓(𝑥1 )


𝑚𝑡𝑎𝑛 = f ‘(x) = 𝑑𝑥 = lim = lim = lim = lim
Δ𝑥 ⟶ 0 Δ𝑥 Δ𝑥 ⟶ 0 Δ𝑥 Δ𝑥 ⟶ 0 Δ𝑥 Δ𝑥 ⟶ 0 Δ𝑥

In general, for any x, we define the slope of a curve as the first derivative of the
function y =f(x) defined by

𝑑𝑦 𝑓(𝑥1 + Δ𝑥)− 𝑓(𝑥1 )


m = f ‘(x) = 𝑑𝑥 = lim Δ𝑥
Δ𝑥 ⟶ 0

LSPU SELF-PACED LEARNING MODULE: TECHNOLOGY FOR TEACHING AND LEARNING


Prepared by: John Louie S. Marasigan
Republic of the Philippines
Laguna State Polytechnic University
Province of Laguna
ISO 9001:2015 Certified
Level I Institutionally Accredited

The Slope of a Curve

The slope m of a curve y =f(x) at (x, f(x)) is equal to the slope of its tangent line
at (x , f(x)) . Hence,
𝑓(𝑥1 + Δ𝑥)− 𝑓(𝑥1 )
m = lim Δ𝑥
Δ𝑥 ⟶ 0
provided that the limit exists.

The Normal Line

The normal line to a curve y = f(x) at a given is the line perpendicular to a tangent
line at that point

The slope of the normal line is equal to the negative reciprocal of the slope of the
tangent line. That is,

1
𝑚𝑛 = −
𝑚𝑡
Rate of Change

In determining the rate of change of one variable with respect to another


variable, we must be able to distinguish between the average and the
instantaneous rates of change. The two rates of change are comparable to the
distinction between the slope of the secant line through two points on a curve
and the slope of the tangent line at one point on the curve.

The Average Rate of Change

The average rate of change of y = f(x) with respect to x in the interval [x, x + Δ𝑥]
is the slope of the secant line joining the points (x , f(x)) and (x + Δ𝑥, f (x + Δ𝑥)) on the
graph of y = f(x).

Δ𝑦 𝑓(𝑥 + Δ𝑥)− 𝑓(𝑥)


Δ𝑥
= Δ𝑥

The Instantaneous Rate of Change

The instantaneous rate of change of y = f(x) with respect to x at the point (x, f(x))
is the slope of the tangent line to the graph of y = f(x) , given by

d𝑦 𝑓(𝑥 + Δ𝑥)− 𝑓(𝑥)


= lim
d𝑥 Δ𝑥 ⟶ 0 Δ𝑥

LSPU SELF-PACED LEARNING MODULE: TECHNOLOGY FOR TEACHING AND LEARNING


Prepared by: John Louie S. Marasigan
Republic of the Philippines
Laguna State Polytechnic University
Province of Laguna
ISO 9001:2015 Certified
Level I Institutionally Accredited

Source Note: Comandante, Felipe L. (2005) Differential Calculus made Easy (Metric Edition) National Bookstore
Average and Instantaneous Rates of Change
Remarks

In future work with derivates, we will use rate of change to mean instantaneous rate of
change

Example: Suppose the function is defined by y=f(x)= 𝑥 2 − 𝑥


a. Find the average rate of change of y with respect to x over the
interval [1 , 3]
b. Find the instantaneous rate of change of y with respect to x at the
point x =-1

Solution

a. From the closed interval [1, 3], we get 𝑥1 = 1 and 𝑥2 = 3. Now, using the formula for
the average rate of change, we get

Δ𝑦 𝑓(𝑥 + Δ𝑥)− 𝑓(𝑥) Δ𝑦 𝑓(𝑥2 )− 𝑓(𝑥1 )


Δ𝑥
= Δ𝑥
⟹ Δ𝑥 = 𝑥2 − 𝑥1

Notice that Δ𝑥 = 𝑥2 - 𝑥1 ⟹ 𝑥2 = 𝑥1 + Δ𝑥

Δ𝑦 𝑓(3)− 𝑓(1) ((3)2− 3) − ( (1)2−1) Δ𝑦


= = ⟹ Δ𝑥 = 3
Δ𝑥 3 −1 3 −1

Thus, on the average y increases 3 units per unit increase in x over the interval [1,3]

b. To find the instrumentation rate of change of y with respect to x, we differentiate the


given function with respect to x. That is,

𝑑𝑦 𝑑𝑦
y = 𝑥2 – 𝑥 ⟹ = 2x -1 ⟹ = -3 at x = -1
𝑑𝑥 𝑑𝑥

Since the instantaneous rate of change is negative at x = -1 , y is decreasing at the point


x = -1

LSPU SELF-PACED LEARNING MODULE: TECHNOLOGY FOR TEACHING AND LEARNING


Prepared by: John Louie S. Marasigan
Republic of the Philippines
Laguna State Polytechnic University
Province of Laguna
ISO 9001:2015 Certified
Level I Institutionally Accredited

Source Note: Comandante, Felipe L. (2005) Differential Calculus made Easy (Metric Edition) National Bookstore
Average and Instantaneous Rates of Change

The Rectilinear Motion

Velocity and acceleration are concepts that every driver of any vehicle
knows. As we drive, the speedometer tells us, at any instant, the velocity of our
vehicle. As we press down on the gas pedal, the velocity of the vehicle changes
and we experience acceleration. In this section, we will analyze our intuitive
notions of velocity and acceleration and replace them with mathematically more
precise concepts. We will show that velocity and acceleration can be calculated
by using derivatives.

Let the position of a vehicle P on a coordinate line l at time t be s(t)


Δ𝑠
i. The average velocity Δ𝑣 of P at time t1 to t2 is Δ𝑣 = Δ𝑡
𝑑𝑠
ii. The instantaneous velocity v(t) of P at time is v(t) = s’(t) = 𝑑𝑡
𝑑𝑠
iii. The speed of P at time t is | v(t) | = | |
𝑑𝑡
𝑑𝑣
iv. The acceleration a(t) of P at time t is a(t) = v’(t) = 𝑑𝑡

Remarks

If an object is acted on only by the force of gravity, and if at time t = 0 , the height
of the object is h0 meters above the ground and the velocity is vo meters per
second, then the object’s height h(t) in meters above the ground at time t seconds
is given by h(t) = -4.9t2 + vot + h0.

LSPU SELF-PACED LEARNING MODULE: TECHNOLOGY FOR TEACHING AND LEARNING


Prepared by: John Louie S. Marasigan
Republic of the Philippines
Laguna State Polytechnic University
Province of Laguna
ISO 9001:2015 Certified
Level I Institutionally Accredited
Limits of a Function

In the study of calculus, the first important concept or idea that must be
introduced is the concept of limit. The limit is the cornerstone of both differential
and integral calculus. It is one of the fundamental ideas that distinguishes
calculus form other areas of mathematics such as algebra or trigonometry.

Now, let us start with an analogy,

Source note: Mathispower4u by James Sousa -Creative Commons Attribution-ShareAlike 3.0


Unported License (n.d.) Retrieved August 20, 2020, from
http://www.mathispower4u.com/

If we want to know the speed of the wind produces by the fan as x gets close to
4, the exact position of the fan. We would measure the wind speed as we get
closer and closer to x=4. The problem is we cannot get the exact position x =4 or
we would risk bodily injury. As a result, we could use the data below to conclude
that the wind speed of the fan at x=4 is approaching 6 mph. This is the idea of a
limit.

Position (x) Speed in mph S(x)


3 5
3.5 5.7
3.9 5.89
3.99 5.985
3.999 5.996

Mathematical we would write this as,

lim 𝑆(𝑥) = 6
𝑥→4

LSPU SELF-PACED LEARNING MODULE: TECHNOLOGY FOR TEACHING AND LEARNING


Prepared by: John Louie S. Marasigan
Republic of the Philippines
Laguna State Polytechnic University
Province of Laguna
ISO 9001:2015 Certified
Level I Institutionally Accredited
Informal definition:

As x approaches a, the limit of f(x) is L , written.

lim 𝑓(𝑥) = 𝐿
𝑥→𝑎
If all values of f(x) are close to L for values of x that are sufficiently close, but not
equal to a.

• Sufficiently close would consist of values that are less (to the left) than
and greater (to the right) than c
• Some functions may have limits at specific values of a
• Notice the value of the limit is not affected by f(a)
Lets consider lim 𝑓(𝑥)
𝑥→1

Source note: Mathispower4u by James Sousa -Creative Commons Attribution-ShareAlike 3.0


Unported License (n.d.) Retrieved August 20, 2020, from
http://www.mathispower4u.com/

x f(x)
0.1 -1.9
0.5 -1.5
0.9 -1.1
0.99 -1.01
0.999 -1.001
1 Undefined
1.0001 -0.999
1.01 -0.99
1.1 -0.9
1.5 -0.9
1.9 -0.1
In general , there are 3 ways to approach finding limits:

• Numerical Approach: t -table


• Graphical Approach: analyze the graph
• Analytical Approach: use algebra or calculus

LSPU SELF-PACED LEARNING MODULE: TECHNOLOGY FOR TEACHING AND LEARNING


Prepared by: John Louie S. Marasigan
Republic of the Philippines
Laguna State Polytechnic University
Province of Laguna
ISO 9001:2015 Certified
Level I Institutionally Accredited

Since we now know about


lim 𝑓(𝑥)
𝑥→𝑎
now is good time to introduce the idea of one-sided limits

lim 𝑓(𝑥) = 𝐿 refers to the limit from the left or values less than a
𝑥 → 𝑎−

lim 𝑓(𝑥) = 𝐿 refers to the limit from the right or values greater than
𝑥 → 𝑎+
a.
Theorem: As x approaches a , the limit of 𝑓(𝑥) = L , if the limit from the left exists
and the limit from the right exists and the limit from the right exists and both
limits are L. That is , if

lim 𝑓(𝑥) = 𝐿 and lim 𝑓(𝑥) = 𝐿


𝑥 → 𝑎− 𝑥 → 𝑎+

then

lim 𝑓(𝑥) = 𝐿
𝑥→𝑎
Consider the limits:

lim 𝑓(𝑥) = −2 lim 𝑓(𝑥) = 3 lim 𝑓(𝑥) = 𝐷𝑁𝐸 (𝐷𝑜𝑒𝑠 𝑛𝑜𝑡 𝑒𝑥𝑖𝑠𝑡)
𝑥 → 1+ 𝑥 → 1− 𝑥→1

Source note: Mathispower4u by James Sousa -Creative Commons Attribution-ShareAlike 3.0


Unported License (n.d.) Retrieved August 20, 2020, from
http://www.mathispower4u.com/

x ( 𝑥 → 1+ ) f(x) x ( 𝑥 → 1− ) f(x)
1.1 -1.9 0.9 2.6
1.01 -1.99 0.99 2.98
1.001 -1.999 0.999 2.998

LSPU SELF-PACED LEARNING MODULE: TECHNOLOGY FOR TEACHING AND LEARNING


Prepared by: John Louie S. Marasigan
Republic of the Philippines
Laguna State Polytechnic University
Province of Laguna
ISO 9001:2015 Certified
Level I Institutionally Accredited
𝑥 2− 1
Example: Evaluate the lim
𝑥 ⟶ 1 𝑥 −1

𝑥 2− 1
Solution: Let f(x) = 𝑥 −1 . Clearly this function is not defined at x=1, that is , f(x)
does not exist. Nevertheless, the function does have a limit as x → 1. Thus,

𝑥 2− 1 (1)2 − 1 0
lim = = 0, it is indeterminate
𝑥 → 1 𝑥 −1 1 −1

We can simplify the expression by factoring the numerator. Hence

𝑥 2− 1 (𝑥 + 1)(𝑥 − 1)
lim = lim = lim (𝑥 + 1) = 1 + 1 = 2
𝑥 → 1 𝑥 −1 𝑥→1 𝑥 −1 𝑥→1
Thus,
𝑥 2− 1
lim =2
𝑥 → 1 𝑥 −1
The graph of the function is the line y = x + 1 , except for a hole at x = 1 .(See
Figure below) . Thus the function is discontinuous at x =1 . Remember that
x → 1 implies that x comes nearer and nearer to 1, but not equal 1. We are only
interested in the behavior of the function in a small neighborhood of x=1, the set
of all nearby points lying to the left of x =1 and to the right but not in what
happens to f(x) at x =1.

Source Note: Comandante, Felipe L. (2005) Differential Calculus made Easy (Metric Edition) National Bookstore
𝑥 2− 1
Figure. Limit of f(x) = as x approaches 1.
𝑥 −1
Remarks:
1. The notation
lim 𝑓(𝑥) = 𝐿
𝑥→𝑎
means that as x gets closer to a but not equal to a., f(x) gets closer to L.

2. The limit of a function as x approaches a is independent of the value of the


function at a even though
lim 𝑓(𝑥)
𝑥→𝑎

exists, the value of the function at a maybe (See Figure below)

LSPU SELF-PACED LEARNING MODULE: TECHNOLOGY FOR TEACHING AND LEARNING


Prepared by: John Louie S. Marasigan
Republic of the Philippines
Laguna State Polytechnic University
Province of Laguna
ISO 9001:2015 Certified
Level I Institutionally Accredited

Source Note: Comandante, Felipe L. (2005) Differential Calculus made Easy (Metric Edition) National Bookstore
Figure : Limit of a Function.

Formal Definition of Limit

Let f(x) be a defined function on an open interval containing a, then

lim 𝑓(𝑥) = 𝐿
𝑥→𝑎

means that for every 𝜀 > 0, there exists 𝛿 > 0 such that

0 < | x -a | < 𝛿 , then | f(x) – L | < 𝜀


(horizontal distance) (vertical distance)

Note: f(x) does not need to be defined at a

This means we need to determine a relationship between 𝛿 and 𝜀 . So for any


vertical distance 𝜀 from L, there is horizontal 𝛿 so that the distance x and a is
less than 𝛿 such that the distance between f(x) and L is less than 𝜀. (See Figure
below)

Source Note: Paul’s Online Notes (2003-2020) Retrieved August 15, 2020, from https://tutorial.math.lamar.edu/

Figure of the Formal Definition of a Limit

LSPU SELF-PACED LEARNING MODULE: TECHNOLOGY FOR TEACHING AND LEARNING


Prepared by: John Louie S. Marasigan
Republic of the Philippines
Laguna State Polytechnic University
Province of Laguna
ISO 9001:2015 Certified
Level I Institutionally Accredited

Properties of Limit

LSPU SELF-PACED LEARNING MODULE: TECHNOLOGY FOR TEACHING AND LEARNING


Prepared by: John Louie S. Marasigan
Republic of the Philippines
Laguna State Polytechnic University
Province of Laguna
ISO 9001:2015 Certified
Level I Institutionally Accredited

Source Note: Paul’s Online Notes (2003-2020) Retrieved August 15, 2020, from https://tutorial.math.lamar.edu/

Example: Compute the value of the following limit


lim (3𝑥 2 + 5𝑥 + 9)
𝑥→2
Solution: Using the properties above
lim (3𝑥 2 + 5𝑥 + 9) = lim 3𝑥 2 + lim 5x - lim 9
𝑥→2 𝑥→2 𝑥→2 𝑥→2

= 3 lim 𝑥 2 + 5 lim x - lim 9


𝑥→2 𝑥→2 𝑥→2

= 3(-2)2 + 5(-2) – 9
= -7
Fact
If p(x) is a polynomial then,
lim 𝑝(𝑥) = 𝑝(𝑎)
𝑥→𝑎

On the above example, 𝑝(𝑥) = 3𝑥 2 + 5𝑥 + 9

Example: Evaluate the following limit


6 − 3𝑥 + 10𝑥 2
lim
𝑥 → 1 −2𝑥 4 + 7𝑥 3 + 1

Solution: Using the properties above,


6 − 3𝑥 + 10𝑥 2 lim 6 − 3𝑥 + 10𝑥 2
lim = 𝑥lim
→1
𝑥 → 1 −2𝑥 4 + 7𝑥 3+1 𝑥→1
−2𝑥 4 + 7𝑥 3+1

6 − 3(1) + 10(1)2
= −2(1)4 + 7(1)3+ 1

13
= 6

LSPU SELF-PACED LEARNING MODULE: TECHNOLOGY FOR TEACHING AND LEARNING


Prepared by: John Louie S. Marasigan
Republic of the Philippines
Laguna State Polytechnic University
Province of Laguna
ISO 9001:2015 Certified
Level I Institutionally Accredited
Example: Compute the value of the following limit

lim [(2𝑥 + 3)(𝑥 2 + 1)]


𝑥→3
Solution:
lim [(2𝑥 + 3)(𝑥 2 + 1)] = lim (2𝑥 + 3) lim (𝑥 2 + 1)
𝑥→3 𝑥→3 𝑥→3

= [2(3) + 3][(3)2 + 3]
= [ 6 + 3 ] [9 + 1]
= 9 [10]
= 90
Fact
Provided f(x) we have,
𝐥𝐢𝐦 𝒇(𝒙) = 𝒇(𝒂) 𝐥𝐢𝐦 𝒇(𝒙) = 𝒇(𝒂) 𝐥𝐢𝐦 𝒇(𝒙) = 𝒇(𝒂)
𝒙→𝒂 𝒙 → 𝒂− 𝒙 → 𝒂+

𝒇(𝒂)
Limit of a Quotient 𝒈(𝒂) where f(a) = 0 and g(a) = 0

In this section, we will discuss some tools in evaluating the limit of a quotient
𝒇(𝒂) 𝒇(𝒙)
in which f(a)=0 and g(a) = 0. The quotient 𝒈(𝒙) is said to assume the
𝒈(𝒂)
0
indeterminate form, at x =a, and it is undefined at that point. Nevertheless, the
0
limit of the quotient exists.

Example: Evaluate the limit

(𝑥 + 𝑎) − 𝑎
lim
𝑥→0 𝑥

Solution: If the denominator consists of a single term and approaches 0,


simplification of the expression should be done first before evaluating the limit.

(𝑥 + 𝑎) −𝑎 (0 + 𝑎) − 𝑎 0
lim = = , it is indeterminate
𝑥→0 𝑥 0 0

We can simplify the expression by removing the grouping symbol. Hence,


(𝑥 + 𝑎) −𝑎 𝑥 + 𝑎 −𝑎 𝑥
lim = lim = lim = lim 1 = 1
𝑥→0 𝑥 𝑥→0 𝑥 𝑥→0𝑥 𝑥→0

LSPU SELF-PACED LEARNING MODULE: TECHNOLOGY FOR TEACHING AND LEARNING


Prepared by: John Louie S. Marasigan
Republic of the Philippines
Laguna State Polytechnic University
Province of Laguna
ISO 9001:2015 Certified
Level I Institutionally Accredited
Example. Evaluate the limit
𝑥2 − 1
lim
𝑥→1 𝑥 − 1

Solution: We factor the numerator or the denominator to simplify the expression


𝑥2 − 1 (1)2 − 1 1−1 0
lim = = = 0 , it is indeterminate
𝑥→1 𝑥 −1 1−1 0

We factor the numerator using the difference of two squares:


(𝑥 2 − 𝑎2 ) = (x + a)(x -a)
𝑥2 − 1 (𝑥 + 1)(𝑥 −1)
lim = lim = lim (x + 1) = 1 + 1 = 2
𝑥→1 𝑥 −1 𝑥→1 𝑥 −1 𝑥→1

Example: Evaluate the limit


𝑥 2 − 5𝑥 + 6
lim
𝑥→3 𝑥 − 3
Solution: We factor the numerator or the denominator to simplify the expression
𝑥 2 −5𝑥 +6 (3)2 − (5)(3) + 6 0
lim = = 0 , it is indeterminate
𝑥→3 𝑥 −3 3−3

Here , we factor the trinomial using the formula:


[𝑥 2 – (a + b)x + ab] = (x – a)(x -b)
𝑥 2 −5𝑥 +6 (𝑥 − 3)(𝑥 − 2)
lim = lim = lim (x – 2) = 3 – 2 = 1
𝑥→3 𝑥 −3 𝑥→3 𝑥−3 𝑥→3

Remarks
Notice that we can shorten this process using
𝑓(x) 𝑓 ′(x)
L’Hopital’s Rule: lim = lim
𝑥 → 𝑎 𝑔(𝑥) 𝑥 → 𝑎 𝑔 ′(𝑥)

G. L’ Hopital was a French nobleman who published the first calculus book. The
rule appeared in that book; however, it was actually discovered by his teacher,
the Swiss mathematician Johann Bernoulli (1667 -1748) , who communicated
the result to G. L’ Hopital in 1684

LSPU SELF-PACED LEARNING MODULE: TECHNOLOGY FOR TEACHING AND LEARNING


Prepared by: John Louie S. Marasigan
Republic of the Philippines
Laguna State Polytechnic University
Province of Laguna
ISO 9001:2015 Certified
Level I Institutionally Accredited
Example: Evaluate the limit
√𝑥 + 2 − √2
lim
𝑥→0 𝑥
Solution: First , we rationalize the numerator then evaluate the limit.
√𝑥 + 2 − √2 √𝑥 + 2 + √2
lim , multiplying the preceding expression by , we get
𝑥→0 𝑥 √𝑥 + 2 + √2

√𝑥 + 2 − √2 √𝑥 + 2 − √2 √𝑥 + 2 + √2
lim = lim . +2 +
𝑥→0 𝑥 𝑥→0 𝑥 √𝑥 √2
(𝑥 + 2) − 2
= lim
𝑥 → 0 𝑥 (√𝑥 + 2 + √2)
1
= lim
𝑥 → 0 √𝑥 + 2 + √2
1 1
= =
√ 2 + √2 2√2

Remarks:
Rationalization is the process of eliminating a radicals or imaginary number from the
denominator of an algebraic fraction. That is, remove the radicals in a fraction so that
the denominator only contains a rational number. (https://www.chegg.com)

One-Sided Limits
In some case a function may approach either of two different limits, depending
on whether the variable approaches its limit through values larger or smaller
that limit. In such a case, the limit is not defined or does not exist but the right-
hand and left-hand limit exists.

The Right-Hand Limit


The right-hand limit L of a function f(x) at a point a is the limit of the function at
a as x approaches a from decreasing values of a , (i.e. as x approaches a from the
right). We write the right-hand limit by the notation

lim 𝑓(𝑥) = 𝐿 or 𝑓(𝑥) → L as x → 𝑎+


𝑥 → 𝑎+
Example. Evaluate the limit of 𝑓(𝑥) = √𝑥 as x approaches 0 from the right
Solution: The graph of 𝑓(𝑥) = √𝑥 is shown in figure below. Notice that the
function is defined only for x ≥ 0 . Now, evaluating the limit of the function as x
approaches 0 from the right, we get

lim √𝑥 = √0 = 0
𝑥 → 0+

LSPU SELF-PACED LEARNING MODULE: TECHNOLOGY FOR TEACHING AND LEARNING


Prepared by: John Louie S. Marasigan
Republic of the Philippines
Laguna State Polytechnic University
Province of Laguna
ISO 9001:2015 Certified
Level I Institutionally Accredited

Source Note: Comandante, Felipe L. (2005) Differential Calculus made Easy (Metric Edition) National Bookstore
Function with a Right – Hand Limit
The Left Hand Limit
The left hand limit L of a function f(x) at a point a is the limit of the function at a
as x approaches a from increasing values of a (i.e. as x approaches a from the
left)
lim− 𝑓(𝑥) = 𝐿 or 𝑓(𝑥) → L as x → 𝑎−
𝑥→𝑎

Example : Given the split function defined by


f(x) = 2 x=1
-2 x<1
Evaluate lim− 𝑓(𝑥) and lim+ 𝑓(𝑥)
𝑥→1 𝑥→1

Solution: The graph of 𝑓(𝑥) is shown in figure below. To evaluate the limit as x
approaches 1 from the left, we use 𝑓(𝑥) = -2. Hence,

lim 𝑓(𝑥) = −2
𝑥 → 1−

Notice that 𝑓(𝑥) = 2for x =1. Now, evaluating the limit as x approaches 1 from
the right, we get
lim+ 𝑓(𝑥) = 𝐷𝑁𝐸 (𝑑𝑜𝑒𝑠 𝑛𝑜𝑡 𝑒𝑥𝑖𝑠𝑡)
𝑥→1

LSPU SELF-PACED LEARNING MODULE: TECHNOLOGY FOR TEACHING AND LEARNING


Prepared by: John Louie S. Marasigan
Republic of the Philippines
Laguna State Polytechnic University
Province of Laguna
ISO 9001:2015 Certified
Level I Institutionally Accredited

Source Note: Comandante, Felipe L. (2005) Differential Calculus made Easy (Metric Edition) National Bookstore
Function with a Left Hand Limit
The Two Sided Limit
The limit exists if and only if the right-hand and left-hand limits both exist and
are equal to L. Thus, we say that the function has a two sided limit

lim 𝑓(𝑥) = 𝐿 ⇔ lim+ 𝑓(𝑥) = lim− 𝑓(𝑥) = 𝐿


𝑥→𝑎 𝑥→𝑎 𝑥→𝑎

Example : Given the sign function


sgn x = 1 x>0
0 x=0
-1 x<0

Evaluate lim− sgn x , lim+ sgn x , lim sgn x


𝑥→0 𝑥→0 𝑥→0

Solution : The graph is shown below

Source Note: Comandante, Felipe L. (2005) Differential Calculus made Easy (Metric Edition) National Bookstore
Function with a One-Sided Limit

LSPU SELF-PACED LEARNING MODULE: TECHNOLOGY FOR TEACHING AND LEARNING


Prepared by: John Louie S. Marasigan
Republic of the Philippines
Laguna State Polytechnic University
Province of Laguna
ISO 9001:2015 Certified
Level I Institutionally Accredited
Evaluating the limit, yields
lim sgn x = - 1 and lim sgn x = 1
𝑥 → 0− 𝑥 → 0+

The limit does not exist since the left hand and right-hand limits are not equal.
Thus,
lim sgn x = does not exist
𝑥→0

Let us look at another kind of problem that can arise in computing some limits
involving piecewise functions.

Example:

Compute the following limits


(a) lim 𝑔(𝑦)
𝑦→6

(b) lim 𝑔(𝑦)


𝑦 → −2

Solution:
(a) lim 𝑔(𝑦)
𝑦→6

lim 𝑔(𝑦) = lim 1 – 3y


𝑦→6 𝑦→6

= - 17
(b) lim 𝑔(𝑦)
𝑦 → −2

In this case the point that we want to take the limit for is the cutoff point for the
two intervals. In other words we can’t just plug y =-2 into the second portion
because this interval does not contain values of y to the left of y = -2 and we need
to know what is happening on both sides of the point.

To do this part we are going to have to remember the fact from the section on
one-sided limits that says that if the two one sided limits exists and are the same
then the normal limit will also exist and have the same value.

Notice that both of the one-sided limits can be done here since we are only going
to be looking at one side of the point in question. So let’s do the two-sided limits
and see what we get.

LSPU SELF-PACED LEARNING MODULE: TECHNOLOGY FOR TEACHING AND LEARNING


Prepared by: John Louie S. Marasigan
Republic of the Philippines
Laguna State Polytechnic University
Province of Laguna
ISO 9001:2015 Certified
Level I Institutionally Accredited
lim 𝑔(𝑦) = lim − 𝑦 2 + 5 since y → −2− implies y < -2
𝑦 → −2− 𝑦 → −2
=9

lim 𝑔(𝑦) = lim + 1 − 3𝑦 since y → −2+ implies y > -2


𝑦 → −2+ 𝑦 → −2
=7
So, in this case we see that,
lim − 𝑔(𝑦)= 9 ≠ 7 = lim + 𝑔(𝑦)
𝑦 → −2 𝑦 → −2

and so since the two one-side limits aren’t the same


lim 𝑔(𝑦) = DNE (does not exist)
𝑦 → −2

Fact
If f(x) ≤ g(x) for all x on [b , d] (except possibly at x =a) and b ≤ a ≤ d then,
lim 𝑓(𝑥) ≤ lim 𝑔(𝑥)
𝑥→𝑎 𝑥→𝑎

So, lim 𝑓(𝑥) = f(a) ≤ g(a) = lim 𝑔(𝑥)


𝑥→𝑎 𝑥→𝑎

The inequality is true because we know that a is somewhere between b and d


and in that range, we also know f(a) ≤ g(a)

Also, note that we said that we assumed that f(a) ≤ g(a) for all x on [b,d] (except
possibly at x =a) . Because limits do not care what is happening at x =a we don’t
really need the inequality to hold at that specific point. We only need it to hold
around x = a since that is what is what the limit is concerned about.

Squeeze Theorem
Suppose that for all x on [b,d] (except possibly at x = a) we have,
f(x) ≤ h(x) ≤ g(x)
Also suppose that ,
lim 𝑓(𝑥) ≤ lim 𝑔(𝑥)
𝑥→𝑎 𝑥→𝑎

for some b ≤ a ≤ d
lim ℎ(𝑥) = 𝐿
𝑥→𝑎

As with the previous fact we only need to know that f(x) ≤ h(x) ≤ g(x) is true
around x = a because we are working with limits and they are only concerned
with what is going on around x =a and not what is actually happening at x = a

LSPU SELF-PACED LEARNING MODULE: TECHNOLOGY FOR TEACHING AND LEARNING


Prepared by: John Louie S. Marasigan
Republic of the Philippines
Laguna State Polytechnic University
Province of Laguna
ISO 9001:2015 Certified
Level I Institutionally Accredited
The following figure illustrates what is happening in this theorem.

Source Note: Paul’s Online Notes (2003-2020) Retrieved August 15, 2020, from https://tutorial.math.lamar.edu/
Squeeze Theorem (Sandwich Theorem or Pinching Theorem)
From the figure we can see that if the limits of f(x)and g(x) are equal at x=a then
the function values must also be equal at x=a .However, because h(x) is
“squeezed” between f(x) and g(x) at this point then h(x) must have the same
value. Therefore, the limit of h(x) at this point must also be the same

Infinite Limits
In this section, we will discuss another case in which a limit fails to exist. We will
discuss functions whose values increase or decrease without bound as the value
of x gets closer and closer to a fixed number.

1
Example: Evaluate the lim , if possible
𝑥 → 1 𝑥 −1
Solution: In figure below , we can see the f(x) decreases without bound as x
approaches 1 from the left and f(x) increases without bound as x approaches 1
from the right
1 1
lim− = - ∞ and lim+ =∞
𝑥 → 1 𝑥 −1 𝑥 → 1 𝑥 −1
Since f(x) is unbounded as x approaches 1, we conclude that the limit does not
exist. Hence,
1
lim = does not exist
𝑥 → 1 𝑥 −1

Source Note: Comandante, Felipe L. Differential Calculus made Easy (Metric Edition) (2005) National Bookstore
Graph of Unbounded Function

LSPU SELF-PACED LEARNING MODULE: TECHNOLOGY FOR TEACHING AND LEARNING


Prepared by: John Louie S. Marasigan
Republic of the Philippines
Laguna State Polytechnic University
Province of Laguna
ISO 9001:2015 Certified
Level I Institutionally Accredited
Remarks
The equal sign in the statement lim 𝑓(𝑥) = ∞ does not mean that the limit
𝑥→𝑎
exists. On the contrary, it tells us how the limit fails to exist by denoting the
unbounded behavior of f(x) as x approaches a.

Notice that the symbol ∞ (or - ∞) does not denote a number but it is used to
describe the situation in which the value of the function is becoming large
without bound (or becoming small without bound)

The Limit Approaching Positive Infinity

The f(x) be a function that is defined at every number in some open interval
containing a except possibly at the number a itself. As x approaches a , f(x)
increases without bound, which is written

lim 𝑓(𝑥) =+ ∞
𝑥→𝑎
if for any number N > 0 there exists a 𝛿 > 0 such that if 0 < | x-a |< 𝛿 then
f(x) > N.
The Limit Approaching Negative Infinity

The f(x) be a function that is defined at every number in some open interval
containing a except possibly at the number a itself. As x approaches a , f(x)
decreases without bound, which is written

lim 𝑓(𝑥) =- ∞
𝑥→𝑎
if for any number N > 0 there exists a 𝛿 > 0 such that if 0 < | x-a |< 𝛿 then
f(x) < N.

Theorem. Let n be any positive integer, then

1
i. lim+ 𝑥 𝑛 = ∞
𝑥→0

1
ii. lim− 𝑥 𝑛 = ∞ if n is even
𝑥→0
-∞ if n is odd
Example: Evaluate the limit
1 1 1
i. lim+ 𝑥 3 ii. lim+ 𝑥 4 iii. lim− 𝑥 3
𝑥→0 𝑥→0 𝑥→0

Solution: Using the Theorem above


1
a. lim+ 𝑥 3 = ∞
𝑥→0

LSPU SELF-PACED LEARNING MODULE: TECHNOLOGY FOR TEACHING AND LEARNING


Prepared by: John Louie S. Marasigan
Republic of the Philippines
Laguna State Polytechnic University
Province of Laguna
ISO 9001:2015 Certified
Level I Institutionally Accredited
1
b. lim+ 𝑥 4 = ∞
𝑥→0
1
c. lim− 𝑥 3 = - ∞
𝑥→0

Facts - Involving Infinite Limits

Given the functions f(x) and g(x) suppose we have

lim 𝑓(x) = ∞ lim 𝑔(x) = 𝐿


𝑥→𝑎 𝑥→𝑎
for some real numbers a and L. Then,

1. lim [ 𝑓(x) ± g(x)] = ∞


𝑥→𝑎

2. If L > 0 then lim [𝑓(x) g(x)] = ∞


𝑥→𝑎

3. If L < 0 then lim [𝑓(x) g(x)] = − ∞


𝑥→𝑎
𝑔(x)
4. lim =0
𝑥 → 𝑎 𝑓(x)

Note as well that the above set of facts also holds for one-sided limits. They will
also hold if 𝐥𝐢𝐦 𝒇(𝐱) = - ∞ with a change of sign on the infinities in the first
𝒙→𝒂
three parts.

Limits at Infinity

In the previous section we saw limits that were infinity and its now time to
look at limits at infinity. By limits at infinity we mean one of the following two
limits.

lim 𝑓(x) lim 𝑓(x)


𝑥→∞ 𝑥→−∞

In other words, we are going to be looking at what happens to a function if we


let x get very large in either the positive or negative sense. Also, as we’ll soon
see, these limits may also have infinity as a value.

Illustration

1
We consider the function 𝑓(x) = 2 . Let x increase without bound ( x → ∞) and
𝑥
let x decrease without bound ( x → − ∞) . Refer to figure below

LSPU SELF-PACED LEARNING MODULE: TECHNOLOGY FOR TEACHING AND LEARNING


Prepared by: John Louie S. Marasigan
Republic of the Philippines
Laguna State Polytechnic University
Province of Laguna
ISO 9001:2015 Certified
Level I Institutionally Accredited

Source Note: Comandante, Felipe L. (2005) Differential Calculus made Easy (Metric Edition) National Bookstore
Figure : Graph of Function where x Increases or decreases without bound.

Analysis
1
We get the following observations from the two tables and graph 𝑓(x) = 𝑥 2
As increases without bound 𝑓(x) approached 0 and also as x decrease without
bound f(x) approaches 0. Now using the concept of limit, we write

1 1
lim =0 and lim =0
𝑥 → ∞ 𝑥2 𝑥 →− ∞ 𝑥 2

The Limit as x Increases Without Bound

Let f(x) be a function that is defined at every number in some interval (a, ∞). The
limit of f(x) as x increases without bound is L, written as

lim 𝑓(x) = 𝐿
𝑥→∞
If for any 𝜀 > 0, however small there exists a number N > 0 such that if x > N then
| f(x) – L | < 𝜀

The Limit as x Decreases Without Bound

Let f(x) be a function that is defined at every number in some interval (- ∞ , a).
The limit of f(x) as x increases without bound is L, written as

lim 𝑓(x) = 𝐿
𝑥 → −∞
If for any 𝜀 > 0, however small there exists a number N < 0 such that if x < N then
| f(x) – L | < 𝜀

LSPU SELF-PACED LEARNING MODULE: TECHNOLOGY FOR TEACHING AND LEARNING


Prepared by: John Louie S. Marasigan
Republic of the Philippines
Laguna State Polytechnic University
Province of Laguna
ISO 9001:2015 Certified
Level I Institutionally Accredited
Horizontal Asymptote
A line y = L is called a horizontal asymptote of the graph of y = f(x) if
lim 𝑓(x) = 𝐿 or lim 𝑓(x) = 𝐿
𝑥→∞ 𝑥 → −∞

Remarks
The horizontal asymptote occurs after evaluating the limit as 𝑥 → ∞
then f(x) → L

Fact

1. If n is a positive rational number and c is any real number then,


𝑐
lim =0
𝑥 → ∞ 𝑥𝑛

2. If n is a positive rational number and c is any real number and 𝑥 𝑛 is defined


for x < 0 then,
𝑐
lim =0
𝑥 → − ∞ 𝑥𝑛

Remarks
To apply the fact above, divide each term of the numerator and denominator by
the variable of the highest power occurring in either the numerator or
denominator.

Example: Evaluate the limit

3𝑥 + 1 2𝑥 3 + 1
i. lim ii. lim
𝑥 → ∞ 4𝑥 − 1 𝑥 → ∞ 1 − 𝑥3
Solution:

i. Dividing each term of the numerator and denominator by x before evaluating


the limit , we obtain
3𝑥 1 1
3𝑥 + 1 + 3+ 3+0 3
lim = lim 𝑥
4𝑥 1
𝑥
= lim 𝑥
1 = =4
𝑥 → ∞ 4𝑥 − 1 𝑥→∞ 𝑥

𝑥
𝑥→∞ 4 −
𝑥
4−0

ii. Dividing each term of the numerator and denominator by 𝑥 3 , yields


1
2𝑥 3 + 1 2+ 3 2+0
lim = lim 1
𝑥
= 0 − 1 = -2
𝑥 → ∞ 1 − 𝑥3 𝑥→∞ −1
𝑥3

LSPU SELF-PACED LEARNING MODULE: TECHNOLOGY FOR TEACHING AND LEARNING


Prepared by: John Louie S. Marasigan
Republic of the Philippines
Laguna State Polytechnic University
Province of Laguna
ISO 9001:2015 Certified
Level I Institutionally Accredited

In the previous section we looked at limits at infinity of polynomials


and/or rational expression involving polynomials. In this section we want to
look at some other types of functions that often show up in limits at infinity. The
functions we will be looking at here are exponentials and natural logarithms.

Let’s start by taking a look at a some of very basic examples involving


exponential functions

Evaluate each of the following limits

lim 𝑒 𝑥 lim 𝑒 𝑥 lim 𝑒 − 𝑥 lim 𝑒 − 𝑥


𝑥→∞ 𝑥→−∞ 𝑥→∞ 𝑥 → −∞

Solution: There are just restatements of facts given in the basic


exponential section,

lim 𝑒 𝑥 = ∞ lim 𝑒 𝑥 = 0 lim 𝑒 −𝑥 = 0 lim 𝑒 −𝑥 = ∞


𝑥→∞ 𝑥→−∞ 𝑥→∞ 𝑥→−∞

Example: Evaluate each of the following limits


2
a.) lim 𝑒 2 − 4𝑥 − 8𝑥
𝑥→∞

4 + 5𝑥 2 + 1
b.) lim 𝑒 𝑥
𝑥→∞

Solution:
2
a.) lim 𝑒 2 − 4𝑥 − 8𝑥 = 0
𝑥→∞

4 + 5𝑥 2 + 1
b.) lim 𝑒 𝑥 = ∞
𝑥→∞

Next, let us take a quick look at some basic limits involving logarithms
Evaluate each of the following limits
lim 𝑙𝑛 𝑥 lim 𝑙𝑛 𝑥
𝑥 → 0+ 𝑥→∞

As with the last example I’ll leave it to verify these statements from the basic
logarithm.

LSPU SELF-PACED LEARNING MODULE: TECHNOLOGY FOR TEACHING AND LEARNING


Prepared by: John Louie S. Marasigan
Republic of the Philippines
Laguna State Polytechnic University
Province of Laguna
ISO 9001:2015 Certified
Level I Institutionally Accredited
lim 𝑙𝑛 𝑥 = - ∞ lim 𝑙𝑛 𝑥 = ∞
𝑥 → 0+ 𝑥→∞

Note that we had to do a right-handed limit for the first one since we can’t plug
negative x’s into a logarithm. This means that the normal limit won’t exist since
we must look at x’s from both sides of the point in question and x’s to the left of
zero are negative.

Example. Evaluate each of the following limits

lim 𝑙𝑛 (7𝑥 3 − 𝑥 2 + 1) = 0
𝑥→∞
Continuity

A function f(x) is said to be continuous at x = a if

lim 𝑓(x) = 𝑓(a)


𝑥→𝑎
A function is said to be continuous on the interval [a , b] if it is continuous at each
point in the interval.

Note that this definition is also implicitly assuming that both f(a) and lim 𝑓(x)
𝑥→𝑎
exist . If their of these do not exist the function will not be continuous at x = a

Fact
• If f(x) is continuous at x = a , then

lim 𝑓(x) = 𝑓(a) lim 𝑓(x) = 𝑓(a) lim 𝑓(x) = 𝑓(a)


𝑥→𝑎 𝑥 → 𝑎− 𝑥 → 𝑎+

• If f(x) is continuous at x = b and lim 𝑔(x) = b then,


𝑥→𝑎

lim 𝑓(𝑔(𝑥)) = 𝑓 ( lim 𝑔(x))


𝑥→𝑎 𝑥→𝑎

Intermediate Value Theorem

Suppose that f(x) is continuous on [b,d] and let M be any number between f(a)
and f(b). Then there exists a number a such that

1. b < a < d
2. f(a) = M

LSPU SELF-PACED LEARNING MODULE: TECHNOLOGY FOR TEACHING AND LEARNING


Prepared by: John Louie S. Marasigan
Republic of the Philippines
Laguna State Polytechnic University
Province of Laguna
ISO 9001:2015 Certified
Level I Institutionally Accredited
Let us go back to the formal definition of limit and use the definition to prove the
limit.

Source Note: Paul’s Online Notes (2003-2020) Retrieved August 15, 2020, from https://tutorial.math.lamar.edu/

Example: Prove the following limit


1
lim ( 𝑥 + 1) = 3
𝑥→6 3

Solution: Let 𝜀 > 0 be any number then we need to find a number 𝛿 > 0 so that
the following will be true.

1
| ( 𝑥 + 1) – 3| < 𝜀 whenever 0<|x–6|<𝛿
3
This gives ,
1
| (3 𝑥 + 1) – 3| < 𝜀
1
⟹ | 3 𝑥 – 2| < 𝜀
1
⟹ 3| 𝑥 – 2| < 3𝜀
3
⟹ | x – 6 | < 3𝜀 = 𝛿

So we may choose, 𝛿 = 3𝜀

Now, let assume that 0 < | x – 6 | < 𝛿 = 3𝜀 . Then,

1 1
| (3 𝑥 + 1) – 3| = |3 𝑥 – 2|
1 6
= |3 x – |
3
1
= | 3 (x – 6) |
1
=3|x-6|

LSPU SELF-PACED LEARNING MODULE: TECHNOLOGY FOR TEACHING AND LEARNING


Prepared by: John Louie S. Marasigan
Republic of the Philippines
Laguna State Polytechnic University
Province of Laguna
ISO 9001:2015 Certified
Level I Institutionally Accredited
Using the assumption | x – 6 | < 𝛿 = 3𝜀
1
< (3𝜀 )
3
=𝜀
Therefore,
1
| (3 𝑥 + 1) – 3| < 𝜀 whenever 0<|x–6|<𝛿
Thus,
1
lim ( 𝑥 + 1) = 3
𝑥→6 3

LSPU SELF-PACED LEARNING MODULE: TECHNOLOGY FOR TEACHING AND LEARNING


Prepared by: John Louie S. Marasigan
Republic of the Philippines
Laguna State Polytechnic University
Province of Laguna
ISO 9001:2015 Certified
Level I Institutionally Accredited

Performance Tasks

Problem Set:

1. Determine the domain and range of the following function using set builder notation

𝑥+1
a.) f(x) = x - 5 b.) f(x) = 𝑥 − 1

1
c.) f(x) = 𝑥 2 + 1 d.) f(x) =
𝑥+1

1
e.) f(x) =
√𝑥

2. Find the limit analytically and support your answer by plotting the graph of the function

a.) lim (2𝑥 + 5)(3𝑥 − 4) b.) lim √1 − 4𝑥


𝑥→5 𝑥→2

𝑥 2 + 2𝑥 + 3 √𝑥 + 5 − 2
c.) lim √ d.) lim
𝑥→2 𝑥2 +5 𝑥 → −1 𝑥+1

4𝑥 + 5
e.) lim
𝑥 → ∞ √2𝑥 2 − 5

3. Use the formal definition of the limit to prove the following limits.

a.) lim (5𝑥 − 𝑥 ) = 3


𝑥→2

b.) lim 𝑥 2 = 16
𝑥→4

c.) lim (𝑥 2 + 𝑥 + 6 ) = 8
𝑥→1

LSPU SELF-PACED LEARNING MODULE: TECHNOLOGY FOR TEACHING AND LEARNING


Prepared by: John Louie S. Marasigan
Republic of the Philippines
Laguna State Polytechnic University
Province of Laguna
ISO 9001:2015 Certified
Level I Institutionally Accredited

LSPU Self-Paced Learning Module (SLM)


Course Calculus
Sem/AY First Semester/2020-2021
Module No. 2nd
Lesson Title The Derivative and Application of Derivatives
Week
5th Week – 9th week
Duration
Date November 3 - December 4, 2020
Description This lesson will discuss rules for finding the derivative, other differentiation rules,
of the higher-order derivatives, implicit differentiation, exponential functions, derivatives of
Lesson exponential functions, logarithmic functions, and derivatives of logarithmic functions.
This lesson also discusses the extrema of functions, applied optimization problems, first
derivative test, concavity and the second derivative test, applied maxima and minima,
and differential.

Learning Outcomes
Intended Students should be able to meet the following intended learning outcomes:
Learning • Write detailed solutions using appropriate mathematical language
Outcomes • Identify areas in Mathematics and other fields where differential calculus is
useful
• Generate solutions to unfamiliar problems and recognize the appropriate tools
of calculus to solve applied problems.

Targets/ At the end of the lesson, students should be able to:


Objectives • Use derivative to find the slope of a curve
• Apply the rules on differentiation to find the derivative of algebraic functions
• Compute higher order derivatives
• Differentiate implicit functions
• Define derivative as a rate of change
• Evaluate the limits of exponential and logarithmic functions
• Find the derivative of exponential and logarithmic functions
• Solve real-world problems involving the limits and derivatives of exponential
and logarithmic functions
• Solve optimization problems involving algebraic relations and functions
• Use the first and second derivative test to solve maxima and minima problems
• Apply the differential of algebraic functions to solve real-world problems

LSPU SELF-PACED LEARNING MODULE: TECHNOLOGY FOR TEACHING AND LEARNING


Prepared by: John Louie S. Marasigan
Republic of the Philippines
Laguna State Polytechnic University
Province of Laguna
ISO 9001:2015 Certified
Level I Institutionally Accredited

Student Learning Strategies


Lecture Guide

In the previous section, we saw that the computation of the slope of a tangent
line, instantaneous rate of change of a function, and the instantaneous velocity
of an object at x =a all required us to compute the following limit.

𝑓(𝑥 + Δ𝑥)− 𝑓(𝑥)


Offline Activities lim
Δ𝑥 ⟶ 0 Δ𝑥
(Self-Paced)Offline
This limit could be written as ,

𝑓(𝑥 + ℎ)− 𝑓(𝑥)


lim
ℎ⟶0 ℎ

Definition of the Derivative

The derivative of f(x) with respect to x is the function f ‘(x) and is defined as,

𝑓(𝑥 + ℎ)− 𝑓(𝑥)


f ‘(x) = lim
ℎ⟶0 ℎ

provided that the limit exists

Remarks:
• We often “read” f ‘(x) as “ f prime of x “
• The process for obtaining the derivative of a function is called derivative
• Various types of notations are used to denote the first derivative of y =
f(x) with respect to x, we have

𝑑𝑦
f ‘(x) = 𝑑𝑥 = Dxy

Example : Find the derivative of the following function using the definition of the
derivative
f(x) = 2𝑥 2 − 16𝑥 + 35

LSPU SELF-PACED LEARNING MODULE: TECHNOLOGY FOR TEACHING AND LEARNING


Prepared by: John Louie S. Marasigan
Republic of the Philippines
Laguna State Polytechnic University
Province of Laguna
ISO 9001:2015 Certified
Level I Institutionally Accredited

Solution: First plug the function into the definition of the derivative

𝑓(𝑥 + ℎ)− 𝑓(𝑥)


f ‘(x) = lim
ℎ⟶0 ℎ
2(𝑥 + ℎ)2 − 16(𝑥 + ℎ) + 35 − (2𝑥 2 − 16𝑥 + 35)
= lim
ℎ⟶0 ℎ
2𝑥 2 + 4𝑥ℎ + 2ℎ 2 − 16𝑥 − 16ℎ + 35 − 2𝑥 2 + 16𝑥 − 35
= lim
ℎ⟶0 ℎ
4𝑥ℎ + 2ℎ2 − 16ℎ
= lim
ℎ⟶0 ℎ
ℎ (4𝑥 + 2ℎ − 16)
= lim
ℎ⟶0 ℎ
= lim 4𝑥 + 2ℎ − 16
ℎ⟶0
= 4x – 16

So, the derivative is

f ‘(x) = 4x – 16

Example. Find the derivative of the following function using the definition of the
derivative
𝑥
g(x) =
𝑥+1
Solution:
𝑔(𝑥 + ℎ)− 𝑔(𝑥)
g ‘(x) = lim
ℎ⟶0 ℎ
1 𝑥 +ℎ 𝑥
= lim -
ℎ⟶0 ℎ 𝑥 +ℎ+1 𝑥+1

1 𝑥 2 + 𝑥 + 𝑥ℎ + ℎ − (𝑥 2 + 𝑥ℎ + 𝑥)
= lim
ℎ⟶0 ℎ ( 𝑥 + ℎ + 1) ( 𝑥 + 1)
1 ℎ
= lim
ℎ ⟶ 0 ℎ ( 𝑥 + ℎ + 1) ( 𝑥 + 1)
1
= lim
ℎ ⟶ 0 ( 𝑥 + ℎ + 1) ( 𝑥 + 1)
1
=
( 𝑥 + 1) ( 𝑥 + 1)
1
=
( 𝑥 + 1)2

So, the derivative is


1
g ‘(x) =
( 𝑥 + 1)2

LSPU SELF-PACED LEARNING MODULE: TECHNOLOGY FOR TEACHING AND LEARNING


Prepared by: John Louie S. Marasigan
Republic of the Philippines
Laguna State Polytechnic University
Province of Laguna
ISO 9001:2015 Certified
Level I Institutionally Accredited

Note:

A function f(x) is called differentiable at x = a if f ‘(a) exists and f (x) is called


differentiable on an interval if the derivative exists for each point in that interval.

Theorem

If f (x) is differentiable at x = a then f (x) is continuous at x =a.

Rules for Finding the Derivative

The previous method for finding the derivative of a function is tedious and
time consuming. Now , we will introduce some rules that provide more
efficient methods in finding the derivative of a function to avoid the
lengthy process.

• The Derivative of a Constant


Let f(x) be a constant function defined by y = f(x) = c is a constant,
then
𝑑𝑦 𝑑
= 𝑑𝑥 (𝑐) = 0
𝑑𝑥

In words, the derivative of a constant is zero

• The Derivative of a Power


Let y = f(x) = 𝑥 𝑛 , where n is positive integer, then for every real
value of x
𝑑𝑦 𝑑
= 𝑑𝑥 (𝑥𝑛 ) = 𝑛𝑥𝑛−1
𝑑𝑥

In words, the derivative of the nth power of a variable is the product


of n and the (n-1) st power of the variable. Or, to differentiate x to a
positive integer power, take the power and multiply it by x to the
next lower integer power.

• The Derivative of a Constant Times a Function


Let c be a constant. If f(x) is a differentiable function at x , then so
is cf(x) and

𝑑𝑦 𝑑 𝑑
= (𝑐 ∙ 𝑓(x) ) = c ∙ [𝑓(x)]
𝑑𝑥 𝑑𝑥 𝑑𝑥

LSPU SELF-PACED LEARNING MODULE: TECHNOLOGY FOR TEACHING AND LEARNING


Prepared by: John Louie S. Marasigan
Republic of the Philippines
Laguna State Polytechnic University
Province of Laguna
ISO 9001:2015 Certified
Level I Institutionally Accredited

• The Derivative of Sums and Differences


Let y = u ± v , where u = 𝑓(x) and v = 𝑔(x) are differentiable
functions of x , then
𝑑𝑦 𝑑 𝑑𝑢 𝑑𝑣
= ( 𝑢 ± 𝑣) = ±
𝑑𝑥 𝑑𝑥 𝑑𝑥 𝑑𝑥

In words, the derivative of a sum of a finite number of differentiable


functions is a sum of the derivative, and the derivatives of the
differences equal the difference of the derivatives.

• The Derivative of a Product


Let y = u ∙ v, where u = 𝑓(x) and v = 𝑔(x) are differentiable
functions of x , then

𝑑𝑦 𝑑 𝑑𝑣 𝑑𝑢
= ( 𝑢 ∙ 𝑣) = 𝑢 ( )+ 𝑣( )
𝑑𝑥 𝑑𝑥 𝑑𝑥 𝑑𝑥

In words, the derivative of a product of two functions is the first


function times the derivative of the second plus the second function
times the derivatives of the first.

• The Derivative of a Quotient


𝑢
Let y = , where u = 𝑓(x) and v = 𝑔(x) are differentiable functions
𝑣
of x and v ≠ 0, then
𝑑𝑢 𝑑𝑣
𝑑𝑦 𝑑 𝑢 𝑣 ( 𝑑𝑥 ) − 𝑢 ( 𝑑𝑥 )
= ( )=
𝑑𝑥 𝑑𝑥 𝑣 𝑣2

In words, the derivative of a quotient of two functions is the


denominator times the derivative of the numerator minus the
numerator times the derivative of the denominator, all divided by
the denominator squared.

• The Derivative of a Reciprocal


1
Let y = , where v = 𝑔(x) are differentiable functions of x and v ≠ 0,
𝑣
then
𝑑𝑢
𝑑𝑦 𝑑 1 (𝑣)
= ( )= 𝑑𝑥
𝑑𝑥 𝑑𝑥 𝑣 𝑣2

In words, the derivative of the reciprocal of a function is the negative of


the derivative of the function divided by the function squared.

LSPU SELF-PACED LEARNING MODULE: TECHNOLOGY FOR TEACHING AND LEARNING


Prepared by: John Louie S. Marasigan
Republic of the Philippines
Laguna State Polytechnic University
Province of Laguna
ISO 9001:2015 Certified
Level I Institutionally Accredited
• The Derivative of a Function with Rational Exponents
We have already established the power formula for n a positive integer.
1
Now, we will extend this formula to rational exponents. Suppose n = 𝑚
1
and y = 𝑥 𝑚 then
1 1
𝑑𝑦 𝑑 1
𝑑𝑥
=
𝑑𝑥
(𝑥 𝑚 ) =
𝑚
𝑥𝑚 − 1

• The Chain Rule


We shall establish a formula for finding the derivative of a composite
function y = f(g(x)). Let y = f(u) and u = g(x), so that y = f(g(x)), then

𝑑𝑦 𝑑 𝑑𝑦 𝑑𝑢
𝑑𝑥
= 𝑑𝑥 (𝑓(𝑔(x))) = 𝑑𝑥 ∙ 𝑑𝑥

In words, if y is a function of u and u is a function of x, then y is a function


of x or a composite function. Hence, the derivative of y with respect to x
is the product of the derivative of y with respect to u and the derivative
of u and the derivative of u with respect to x.

• The General Power Rule


Let y = [𝑢(𝑥)]𝑛 , where u = 𝑓(x) is differentiable function of x and n
is a real number, then
𝑑𝑦 𝑑 𝑛 𝑛 − 1 𝑑𝑢
= 𝑑𝑥 [𝑢(𝑥)] = 𝑛 [𝑢(𝑥)]
𝑑𝑥 𝑑𝑥

In words the derivative of the nth power of a differentiable


function is the product of n, the (n – 1)st power of the function and
the derivative of the function.

Example: Differentiate each of the following functions

a.) 𝑓(x) = 15𝑥 100 − 3𝑥 12 + 5𝑥 − 46

Solutions:
𝑓′(x) = (100) 15𝑥 99 − (12)3𝑥11 + (1)5𝑥 0 – 0
= 1500𝑥 99 - 36𝑥11 + 5

LSPU SELF-PACED LEARNING MODULE: TECHNOLOGY FOR TEACHING AND LEARNING


Prepared by: John Louie S. Marasigan
Republic of the Philippines
Laguna State Polytechnic University
Province of Laguna
ISO 9001:2015 Certified
Level I Institutionally Accredited
b.) g(t) = 2𝑡 6 + 7𝑡 −6

Solution:
g’(t) = (6)2𝑡 5 + (−6) 7𝑡 −7
= 12𝑡 5 - 42𝑡 −7
42
= 12𝑡 5 - 𝑡 7
1
c.) 𝑓(x) = 8𝑥 3 - 3 𝑥 −5 + 𝑥 – 23

Solution:
1
𝑓′(x) = (3) 8𝑥 2 – (-5) 3 𝑥 −6 + (1)𝑥 0 – 0
5
= 24𝑥 2 + 3 𝑥 −6 + 1
5
= 24𝑥 2 + 3𝑥 6 + 1

3 2
d.) 𝑓(x) = √𝑥 + 9√𝑥 7 - 5
√𝑥 2

Solution:
1 1 7 4 2 7
𝑓′(x) = 𝑥 − 2 + 9( ) 𝑥 3 – 2 (− ) 𝑥 −5
2 3 5

1 1 63 4 4 7
= 𝑥− 2 + 𝑥3 + 𝑥 −5
2 3 5

e.) 𝑓(x) = (6𝑥 3 − 𝑥)(10 − 20𝑥)

Solution:
𝑓′(x) = (10 − 20𝑥) (18𝑥 2 − 1) + (6𝑥 3 − 𝑥)(-20)
= -480𝑥 3 + 180𝑥 2 + 40x – 10
4√𝑥
f.) h(x) = 𝑥 2 − 2

Solution:
1 1
1 −
4( )𝑥 2 (𝑥 2 − 2) − 4𝑥 2 (2𝑥)
h’(x) = 2
(𝑥 2 − 2)2
3 1

− 6𝑥 2 − 4𝑥 2
= (𝑥 2 − 2)2

LSPU SELF-PACED LEARNING MODULE: TECHNOLOGY FOR TEACHING AND LEARNING


Prepared by: John Louie S. Marasigan
Republic of the Philippines
Laguna State Polytechnic University
Province of Laguna
ISO 9001:2015 Certified
Level I Institutionally Accredited
Higher Order Derivatives

The derivative of a function is a function itself. We may consider finding the


derivative of the first derivative that is the second derivative and the derivative
of the second derivative that is the third derivative and so on,

High order Derivatives are the result of two or more successive


differentiations of a function. For instance, given the function y = f(x) we may
define the higher-order derivatives as follows

𝑑𝑦
= f ‘(x). . . first derivative
𝑑𝑥

𝑑2 𝑦 𝑑 𝑑𝑦
=𝑑𝑥 (𝑑𝑥 ) = f ‘’(x). . . second derivative
𝑑𝑥 2
.
.
.
𝑑𝑛 𝑦 𝑑 𝑑𝑛−1𝑦
= ( ) = 𝑓 𝑛 (x). . . nth derivative
𝑑𝑥 𝑛 𝑑𝑥 𝑑𝑥 𝑛−1

Remarks
So long as each successive derivative is differentiable, we can continue in this
manner to obtain the derivatives of higher order.

Example: Find the first, second and third derivatives of

𝑓(x) = 𝑥 3 − 2𝑥 −3 − 4𝑥 + 2

Solution:
𝑑𝑦
= 5𝑥 4 + 6𝑥 −4 – 4
𝑑𝑥

𝑑2 𝑦
= 20𝑥 3 + 24𝑥 −5
𝑑𝑥 2

𝑑3 𝑦
= 60𝑥 2 + 120𝑥 −6
𝑑𝑥 3

LSPU SELF-PACED LEARNING MODULE: TECHNOLOGY FOR TEACHING AND LEARNING


Prepared by: John Louie S. Marasigan
Republic of the Philippines
Laguna State Polytechnic University
Province of Laguna
ISO 9001:2015 Certified
Level I Institutionally Accredited
Implicit Differentiation

In the preceding sections, we discussed functions in the form of y = f(x) which


expresses y explicitly in terms of x and differentiated according to the rules for
the types of functions involved. However, some do not give y explicitly in terms
to x . Such equation is said to determine y as an implicit function of x , that is of
the form f(x) = 0.

To find the derivative of an implicit function, f(x) = 0 with respect to x , y is


considered as an unknown but differentiable function of x , and every time we
𝑑𝑦
differentiate y, we differentiate it as usual but multiply the result by 𝑑𝑥 .

𝑑𝑦
Example: Find 𝑑𝑥 , if xy = x + y

Solution:
xy = x + y
𝑑𝑦 𝑑𝑦
x(1) ∙ + y(1) = (1) + (1) ∙
𝑑𝑥 𝑑𝑥
𝑑𝑦 𝑑𝑦
x∙ + y = 1 + 𝑑𝑥
𝑑𝑥
𝑑𝑦
𝑑𝑥
∙(x – 1) = 1 – y
𝑑𝑦 1–y x
= x – 1 , since y = x – 1
𝑑𝑥
𝑥
𝑑𝑦 1–x–1
=
𝑑𝑥 x–1
𝑑𝑦 1
=− 2
𝑑𝑥 (𝑥 −1)
𝑑𝑦
Example: Find 𝑑𝑥 , if √𝑥 + 𝑦 + 1 = 𝑥 2 − 𝑦 2

Solution:
√𝑥 + 𝑦 + 1 = 𝑥 2 − 𝑦 2
1
(𝑥 + 𝑦)2 + 1 = 𝑥 2 − 𝑦 2
1 𝑑𝑦
1 𝑑𝑦
(𝑥 + 𝑦)− 2 (1+ 𝑑𝑥 ) + 0 = 2x – 2y ∙
2 𝑑𝑥
1 1 𝑑𝑦
− − 𝑑𝑦
(𝑥 + 𝑦) 2 + (𝑥 + 𝑦) 2 ∙ 𝑑𝑥 = 4x – 4y ∙
𝑑𝑥

LSPU SELF-PACED LEARNING MODULE: TECHNOLOGY FOR TEACHING AND LEARNING


Prepared by: John Louie S. Marasigan
Republic of the Philippines
Laguna State Polytechnic University
Province of Laguna
ISO 9001:2015 Certified
Level I Institutionally Accredited

𝑑𝑦 1 1
+ 4y = 4x -
𝑑𝑥 √𝑥 + 𝑦 √𝑥 + 𝑦
4𝑥√𝑥 + 𝑦 − 1
𝑑𝑦 √𝑥 + 𝑦
= 1 + 4𝑦 √𝑥 + 𝑦
𝑑𝑥
√𝑥 + 𝑦

𝑑𝑦 4𝑥 𝑥 + 𝑦 − 1
= 4𝑦√𝑥 + 𝑦 + 1
𝑑𝑥 √

Derivative of Exponential Functions


Remarks: A function in which an unknown quantity appears in one or more
exponent is called an exponential function.

In this section lets review again the behavior of the exponential functions, limit
of exponential function and solve applied problems involving exponential
functions.

The Exponential Functions

If a > 0 and a ≠ 1 , then the exponential function with base a is

𝑦 = 𝑓(𝑥) = 𝑎 𝑥

where 𝑓(𝑥) = 𝑎 𝑥 is always positive. Its graph always lies above the x-axis

Since f (0) = 1, the graph always passes through the point (0,1). If a > 1, the
function is an increasing function. If 0 < a < 1, the function is a decreasing
function. The curve is concave upward for a > 1 and for 0 < a < 1. The domain
consists of all real numbers x and the range consist of all positive numbers y. The
negative x-axis is the horizontal asymptote for a > 1 and the positive x – axis is
the horizontal asymptote for 0 < a < 1.

Example. Sketch the graph y = 𝑎 𝑥 both for a > 1 and 0 < a < 1

Solution
Notice that the x – axis is horizontal asymptote (See Figure below)

For y = 𝑎 𝑥 , a > 1 For y = 𝑎 𝑥 , 0 < a < 1


Domain: { x | x ∈ ℝ } Domain: { x | x ∈ ℝ }
Range: {y| y ∈ ℝ , y > 0 } Range: {y| y ∈ ℝ , y > 0 }
Horizontal Asymptote: Negative x-axis Horizontal Asymptote: Positive x-axis

LSPU SELF-PACED LEARNING MODULE: TECHNOLOGY FOR TEACHING AND LEARNING


Prepared by: John Louie S. Marasigan
Republic of the Philippines
Laguna State Polytechnic University
Province of Laguna
ISO 9001:2015 Certified
Level I Institutionally Accredited

Source Note: Comandante, Felipe L. (2005) Differential Calculus made Easy (Metric Edition) National Bookstore
Graphs of y = 𝑎 𝑥
The Limit of y = 𝑎 𝑥

The limit of y = 𝑎 𝑥 , a > 1 is


• lim+ 𝑎 𝑥 = 1
𝑥→0

• lim 𝑎 𝑥 = 1
𝑥 → 0−

• lim 𝑎 𝑥 = ∞
𝑥→∞

• lim 𝑎 𝑥 = 0
𝑥 → −∞

The limit of y = 𝑎 𝑥 , 0 < a < 1 is


• lim+ 𝑎 𝑥 = 1
𝑥→0

• lim 𝑎 𝑥 = 1
𝑥 → 0−

• lim 𝑎 𝑥 = 0
𝑥→∞

• lim 𝑎 𝑥 = ∞
𝑥 → −∞

The Natural Number e


In calculus, the natural number e which is an irrational number is the convenient
choice for a base, whose decimal approximation is given by

e ≈ 2.71828182846

However, the convenience of this choice will become evident when we develop
the rules for differentiation of exponential and logarithmic function. Now, we
will establish the limit definition of the number e.
1
e = lim (1 + 𝑥)𝑥
x⟶0

LSPU SELF-PACED LEARNING MODULE: TECHNOLOGY FOR TEACHING AND LEARNING


Prepared by: John Louie S. Marasigan
Republic of the Philippines
Laguna State Polytechnic University
Province of Laguna
ISO 9001:2015 Certified
Level I Institutionally Accredited
Example Sketch the graphs of y = 𝑒 𝑥 and y = 𝑒 −𝑥 . Determine the asymptote,
domain and range

Solution:
For y = 𝑒 𝑥 For y = 𝑒 −𝑥
Domain: {x | x ∈ ℝ } Domain: {x | x ∈ ℝ }
Range: {y| y ∈ ℝ , y > 0} Range: {y| y ∈ ℝ , y > 0 }
Horizontal Asymptote: Negative x-axis Horizontal Asymptote: Positive x-axis

Table

x -3 -2 -1 0 1 2 3
y = 𝑒𝑥 0.0498 0.1353 0.3679 1 2.7183 7.3891 20.0855
y = 𝑒 −𝑥 20.0855 7.3891 2.7183 1 0.3679 0.1353 0.0498

Source Note: Comandante, Felipe L. (2005) Differential Calculus made Easy (Metric Edition) National Bookstore
Graphs of y = 𝑒 𝑥 and y = 𝑒 −𝑥

The Limit of y = 𝑒 𝑥 and y = 𝑒 −𝑥

The limit of y = 𝑒 𝑥 , is
• lim+ 𝑒 𝑥 = 1
𝑥→0

• lim 𝑒 𝑥 = 1
𝑥 → 0−

• lim 𝑒 𝑥 = ∞
𝑥→∞

• lim 𝑒 𝑥 = 0
𝑥 → −∞

The limit of y = 𝑒 𝑥 ,is


• lim+ 𝑒 𝑥 = 1
𝑥→0

• lim 𝑒 𝑥 = 1
𝑥 → 0−

• lim 𝑒 𝑥 = 0
𝑥→∞

• lim 𝑒 𝑥 = ∞
𝑥 → −∞

LSPU SELF-PACED LEARNING MODULE: TECHNOLOGY FOR TEACHING AND LEARNING


Prepared by: John Louie S. Marasigan
Republic of the Philippines
Laguna State Polytechnic University
Province of Laguna
ISO 9001:2015 Certified
Level I Institutionally Accredited

Derivatives of Exponential Functions

We will derive the formula for finding the derivative of y = f(x) = ex using the
limit definition of derivative. Notice that for any function y = f(x)
𝑓(𝑥 + Δ𝑥)− 𝑓(𝑥)
y = f(x) = lim (1)
Δ𝑥 ⟶ 0 Δ𝑥
This limit could be written as,
𝑓(𝑥 + ℎ)− 𝑓(𝑥)
y = f(x) = lim
ℎ⟶0 ℎ
If y = f(x) = 𝑒 , we can find its derivative using (1). Hence,
𝑥

𝑑𝑦 𝑑 𝑒𝑥 + Δ𝑥 − 𝑒𝑥 𝑑 𝑒𝑥 (𝑒Δ𝑥 −1)
= 𝑑𝑥 (𝑒 𝑥 ) = lim ⟹ (𝑒 𝑥 ) = lim
𝑑𝑥 Δ𝑥 ⟶ 0 Δ𝑥 𝑑𝑥 Δ𝑥 ⟶ 0 Δ𝑥

From the definition of the number 𝑒 , we replace x with Δ𝑥,


1 1
e = lim (1 + 𝑥)𝑥 ⟹ e = lim (1 + Δ𝑥)Δ𝑥
x⟶0 Δ𝑥⟶ 0

Now, for small values of Δ𝑥 , we obtain


1
e = (1 + Δ𝑥)Δ𝑥 ⟹ eΔ𝑥 = (1 + Δ𝑥)
By substitution,
𝑑 𝑒𝑥 (1+Δ𝑥 −1) 𝑑
(𝑒 𝑥 ) = lim ⟹ (𝑒𝑥 ) = 𝑒 𝑥
𝑑𝑥 Δ𝑥 ⟶ 0 Δ𝑥 𝑑𝑥

In general, let u = f(x), so that


𝑑 𝑑𝑢
(𝑒 𝑢 ) = 𝑒 𝑢 𝑑𝑥
𝑑𝑥

Example: Find the first derivative


2+2𝑥−1
y = 𝑒 3𝑥

𝑑 𝑑𝑢
Solution: We use formula 𝑑𝑥 (𝑒 𝑢 ) = 𝑒 𝑢 𝑑𝑥 to find the first derivative

Let u = 3𝑥 2 + 2𝑥 − 1
𝑑𝑦 𝑑 2 +2𝑥−1 2+2𝑥−1 2+2𝑥−1
= 𝑑𝑥 (𝑒 3𝑥 ) = 𝑒 3𝑥 (6x + 2) =2(3x + 1) 𝑒 3𝑥
𝑑𝑥
𝑑 2+2𝑥−1 2 +2𝑥−1
(𝑒 3𝑥 ) = 2(3x + 1) 𝑒 3𝑥
𝑑𝑥

LSPU SELF-PACED LEARNING MODULE: TECHNOLOGY FOR TEACHING AND LEARNING


Prepared by: John Louie S. Marasigan
Republic of the Philippines
Laguna State Polytechnic University
Province of Laguna
ISO 9001:2015 Certified
Level I Institutionally Accredited
Logarithmic Functions
Logarithms were invented in the seventeenth century and until the recent years
they were an important tool in arithmetic calculation. However, with the
existence of calculators and computers, the application of logarithms is less
important. But the principle of logarithms and their applications are still useful
in calculus.

The Logarithmic Function y = 𝑙𝑜𝑔𝑎 𝑥

For any positive number a ≠ 1, the logarithmic function to the base a is the
inverse of the exponential function to the base a , we write

y = 𝑙𝑜𝑔𝑎 𝑥 if and only if 𝑎𝑦 = x

It is an increasing function if a > 1 and decreasing if 0 < a < 1. As x increases


through positive values, log x increases more and more slowly. The domain is
the set of positive real numbers while its range is the set of all real numbers. The
graph intercepts the x-axis at x = 1, thus if x = 1, then y = 0. If x = 0, then the
function is not defined. The logarithm of a negative number is not on the graph
of real numbers.

Example Sketch the graphs of y = 𝑙𝑜𝑔𝑎 𝑥 and y= 𝑎 𝑥 on the same axis


Solutions: The graph of y = 𝑙𝑜𝑔𝑎 𝑥 and y= 𝑎 𝑥 are shown across the line y = x
since the two functions are inverse (See Figure below)

For y = 𝑙𝑜𝑔𝑎 𝑥 For y = 𝑎 𝑥


Domain: { x | x ∈ ℝ , 𝑥 > 0 } Domain: { x | x ∈ ℝ }
Range: {y| y ∈ ℝ , y > 0 } Range: {y| y ∈ ℝ , y > 0 }

Graphs of y = 𝑎 𝑥 and y = 𝑙𝑜𝑔𝑎 𝑥

Source Note: Comandante, Felipe L. (2005) Differential Calculus made Easy (Metric Edition) National Bookstore

LSPU SELF-PACED LEARNING MODULE: TECHNOLOGY FOR TEACHING AND LEARNING


Prepared by: John Louie S. Marasigan
Republic of the Philippines
Laguna State Polytechnic University
Province of Laguna
ISO 9001:2015 Certified
Level I Institutionally Accredited
The Natural Logarithmic Function
The natural logarithmic function is y = ln x if and only if 𝑒 𝑦 = 𝑥
Example: Sketch the graph of y = ln x and 𝑦 = 𝑒 𝑥 on the same axis.
Solution: The graphs of the natural logarithmic function y = ln x and the natural
exponential function 𝑦 = 𝑒 𝑥 (See Figure Below). The two functions are
symmetric about the line y =x because they are inverse. The domain of y = ln x is
the set of real numbers greater than zero and the range is also the set real
numbers

Domain: {x | x ∈ ℝ , 𝑥 > 0 } Domain: { x | x ∈ ℝ }

Graphs of y = ln x and 𝑦 = 𝑒 𝑥

Source Note: Comandante, Felipe L. (2005) Differential Calculus made Easy (Metric Edition) National Bookstore

The Limit of y = ln x

The limit of y = ln x is

i. lim ln 𝑥 = − ∞ ii . lim ln 𝑥 = ∞
𝑥 → 0+ 𝑥 →+ ∞

Derivatives of Logarithmic Functions

We will derive the formula for finding the derivative of the natural logarithmic
function using the inverse relations of 𝑒 𝑥 and ln x. To do this, we let

𝑦 = ln 𝑥

Taking the inverse, we obtain

𝑒 𝑦 = 𝑒 𝑙𝑛 𝑥 ⇒ 𝑒𝑦 = 𝑥

LSPU SELF-PACED LEARNING MODULE: TECHNOLOGY FOR TEACHING AND LEARNING


Prepared by: John Louie S. Marasigan
Republic of the Philippines
Laguna State Polytechnic University
Province of Laguna
ISO 9001:2015 Certified
Level I Institutionally Accredited
𝑑𝑦
Differentiating implicitly then solving for 𝑑𝑥 , we get

𝑑𝑦 𝑑𝑦 1
𝑒𝑦 ∙ =1 ⇒ = 𝑦
𝑑𝑥 𝑑𝑥 𝑒

Substituting 𝑒 𝑦 = 𝑥, we have

𝑑𝑦 1 𝑑 1
= ⇒ (ln 𝑥) =
𝑑𝑥 𝑥 𝑑𝑥 𝑥

In general, let 𝑢 = 𝑓(𝑥) so that

𝑑 1 𝑑𝑢
(ln 𝑢) =
𝑑𝑥 𝑢 𝑑𝑥
Using differential, we get

𝑑𝑢
𝑑(ln 𝑢) =
𝑢
𝑑𝑦
Example: Find , if
𝑑𝑥

a. 𝑦 = 𝑙𝑛√𝑥 2 + 1 b. 𝑦 = ln 𝑥 𝑥

Solution:

We use the formula


𝑑𝑢
𝑑(ln 𝑢) =
𝑢
to differentiate each function

a.) Given 𝑦 = 𝑙𝑛√𝑥 2 + 1 ,

𝑑𝑢 𝑥
Let 𝑢 = √𝑥 2 + 1 , so that = √𝑥 2 . Hence
𝑑𝑥 +1

𝑑𝑦 𝑑 1 𝑥 𝑥
= (𝑙𝑛√𝑥2 + 1) = =
𝑑𝑥 𝑑𝑥 √𝑥2 +1 √𝑥2 +1 𝑥2 +1

LSPU SELF-PACED LEARNING MODULE: TECHNOLOGY FOR TEACHING AND LEARNING


Prepared by: John Louie S. Marasigan
Republic of the Philippines
Laguna State Polytechnic University
Province of Laguna
ISO 9001:2015 Certified
Level I Institutionally Accredited

b.) Given 𝑦 = ln 𝑥 𝑥 ,

We can simplify 𝑦 = ln 𝑥 𝑥 into 𝑦 = 𝑥 𝑙𝑛 𝑥 . Using the product rule, we


get

𝑑𝑦 𝑑 𝑑
=𝑥 (ln 𝑥) + ln 𝑥 (𝑥)
𝑑𝑥 𝑑𝑥 𝑑𝑥
𝑑𝑦 1
=𝑥 + (1) ln 𝑥
𝑑𝑥 𝑥

𝑑
(ln 𝑥 𝑥 ) = 1 + ln 𝑥
𝑑𝑥

The Constant with a Variable Exponent

If a is any positive number and x is any real number, then we define

𝑎 𝑥 = 𝑒 𝑥 𝑙𝑛 𝑎

The Derivative of a Constant with a Variable Exponent

Now, we will derive a formula for finding the derivative of functions whose
value is a constant with a variable exponent, 𝑦 = 𝑎 𝑥 . We write 𝑦 = 𝑎 𝑥 in
terms of natural exponent and obtain

𝑦 = 𝑎 𝑥 = 𝑒 𝑥 ln 𝑎

Differentiating y with respect to x using the formula: 𝑑 𝑒 𝑢 = 𝑒 𝑢 𝑑𝑢 , yields

𝑑 𝑥
(𝑎 ) = (𝑒 𝑥 ln 𝑎 )(ln 𝑎)
𝑑𝑥

Substituting 𝑎 𝑥 = 𝑒 𝑥 ln 𝑎 , we obtain
𝑑
(𝑎 𝑥 ) = 𝑎 𝑥 ln 𝑎
𝑑𝑥
𝑑 𝑑𝑢
In general, let 𝑢 = 𝑓(𝑥), so that 𝑑𝑥 (𝑎𝑢 ) = 𝑎 𝑥 ln 𝑎 (𝑑𝑥 )

Using differentials, we get 𝑑 (𝑎𝑢 ) = (𝑎𝑢 ln 𝑎)𝑑𝑢

LSPU SELF-PACED LEARNING MODULE: TECHNOLOGY FOR TEACHING AND LEARNING


Prepared by: John Louie S. Marasigan
Republic of the Philippines
Laguna State Polytechnic University
Province of Laguna
ISO 9001:2015 Certified
Level I Institutionally Accredited
𝑑𝑦
Example: Find 𝑑𝑥 , if

2−2𝑥+1
a. 𝑦 = 23𝑥+4 b. 𝑦 = ln 3𝑥

𝑑𝑦
Solution : We use the formula: 𝑑 (𝑎𝑢 ) = (𝑎𝑢 𝑙𝑛 𝑎)𝑑𝑢 to find 𝑑𝑥

a. 𝑦 = 23𝑥+4

𝑑𝑢
Let 𝑢 = 3𝑥 + 4 , so that 𝑑𝑥 = 3. Hence

𝑑𝑦 𝑑 3𝑥+4
= (2 ) = 23𝑥+4 3 ln 2
𝑑𝑥 𝑑𝑥
2−2𝑥+1
b. y = ln 3𝑥
2−2𝑥+1 𝑑𝑢 2−2𝑥+1
Let 𝑢 = 3𝑥 , so that 𝑑𝑥 = 3𝑥 (2𝑥 − 2) ln 3

2−2𝑥+1
𝑑𝑦 𝑑 𝑥 2−2𝑥+1
3𝑥 (2𝑥 − 2) ln 3
= (ln 3 )= 2−2𝑥+1
𝑑𝑥 𝑑𝑥 3𝑥

The Derivative of a Function with a Function as its Exponent

We will derive the formula for finding the derivative of a function with a
function as its exponent. Let u and v be functions of x so that 𝑦 = 𝑢𝑣 . We
differentiate 𝑦 = 𝑢𝑣 with respect to x using two methods.

Method 1 We express the function in terms of the exponent of a natural


number and obtain

𝑦 = 𝑢𝑣 ⇒ 𝑦 = 𝑒 𝑣 ln 𝑢

Differentiating y with respect to x, we get

𝑑𝑦 1 𝑑𝑢 𝑑𝑣
= 𝑒 𝑣 ln 𝑢 [ 𝑣 ∙ ( ) + ln 𝑢 ( )]
𝑑𝑥 𝑢 𝑑𝑥 𝑑𝑥

𝑑𝑦 𝑣 𝑑𝑢 𝑑𝑣
= 𝑒 𝑣 ln 𝑢 [ ( ) + ln 𝑢 ( )]
𝑑𝑥 𝑢 𝑑𝑥 𝑑𝑥

LSPU SELF-PACED LEARNING MODULE: TECHNOLOGY FOR TEACHING AND LEARNING


Prepared by: John Louie S. Marasigan
Republic of the Philippines
Laguna State Polytechnic University
Province of Laguna
ISO 9001:2015 Certified
Level I Institutionally Accredited
Substituting 𝑦 = 𝑒 𝑣 ln 𝑢 , we have

𝑑𝑦 𝑑𝑢 𝑑𝑣
= v𝑢𝑣−1 ∙ 𝑑𝑥 + 𝑢𝑣 ln 𝑢 ∙ 𝑑𝑥
𝑑𝑥

Method 2 We use logarithmic differentiation to find the derivative of 𝑦 = 𝑢𝑣

Taking the natural logarithm of both sides, we obtain

ln 𝑦 = ln 𝑢𝑣 ⇒ ln 𝑦 = v ln 𝑢

Differentiating implicitly with respect to x, we get

1 𝑑𝑦 1 𝑑𝑢 𝑑𝑣 𝑑𝑦 𝑑𝑢 𝑑𝑣
=𝑣 + ln 𝑢 ⇒ = 𝑦 [𝑣𝑢−1 + ln 𝑢 ]
𝑦 𝑑𝑥 𝑢 𝑑𝑥 𝑑𝑥 𝑑𝑥 𝑑𝑥 𝑑𝑥

Substituting 𝑦 = 𝑢𝑣 , yields

𝑑𝑦 𝑑𝑢 𝑑𝑣
= 𝑣𝑢−1 + ln 𝑢
𝑑𝑥 𝑑𝑥 𝑑𝑥
𝑑𝑦
Example: Find 𝑑𝑥 , if 𝑦 = 𝑥 √𝑥

Solution:

Given 𝑦 = 𝑥 √𝑥

Method 1 We write 𝑦 = 𝑥 √𝑥 in terms of natural exponent, we get

𝑦 = 𝑒 √𝑥 ln 𝑥
𝑑 𝑑𝑢
Differentiating y with respect to x using the formula: 𝑑𝑥 (𝑒 𝑢 ) = 𝑒 𝑢 𝑑𝑥 , yields
𝑑𝑦 √𝑥 ln 𝑥
= 𝑒 √𝑥 ln 𝑥 ( 𝑥 + 2 )
𝑑𝑥 √𝑥

Substituting 𝑦 = 𝑥 √𝑥 , we have

𝑑𝑦 𝑥 √𝑥
= (2 + ln 𝑥)
𝑑𝑥 2 √𝑥

LSPU SELF-PACED LEARNING MODULE: TECHNOLOGY FOR TEACHING AND LEARNING


Prepared by: John Louie S. Marasigan
Republic of the Philippines
Laguna State Polytechnic University
Province of Laguna
ISO 9001:2015 Certified
Level I Institutionally Accredited

Method 2 We use the method of logarithmic differentiation to find the


derivative of 𝑦 = 𝑥 √𝑥

Taking the ln of both sides, we obtain

ln 𝑦 = ln 𝑥 √𝑥 ⇒ ln 𝑦 = √𝑥 ln 𝑥

Differentiating implicitly with respect to x, we get

1 𝑑𝑦 1 1 𝑑𝑦 √𝑥 ln 𝑥
= √𝑥 + ln 𝑥 ( ) ⇒ = 𝑦( + )
𝑦 𝑑𝑥 𝑥 2 √𝑥 𝑑𝑥 𝑥 2√𝑥

Substituting 𝑦 = 𝑥 √𝑥

𝑑𝑦 𝑥 √𝑥
= ( 2 + ln 𝑥)
𝑑𝑥 2 √𝑥

Related Rates
In this section we will consider problems involving two (or more) variables that
are changing with respect to time t . In such problems we want to find the rate
at which some quantity is changing by relating it to other quantities whose rates
of change are known. If two such quantities are related to each other, then their
rates of change with respect to time t are also related.

For instance, if x and y are related by the equation 𝑥 2 + 𝑦 2 = 1 then differentiating


this equation with respect to t produces an equation involving the rates of
𝑑𝑥 𝑑𝑦
change 𝑑𝑡 and 𝑑𝑡 . Hence,

𝑑𝑥 𝑑𝑦 𝑑𝑦 𝑥 𝑑𝑥
2x 𝑑𝑡 + 2y 𝑑𝑡 = 0 ⟹ = −𝑦 ∙
𝑑𝑡 𝑑𝑡

𝑑𝑥 𝑑𝑦
The derivatives 𝑑𝑡 and 𝑑𝑡 are called related rates since they are related by means
of an equation.

LSPU SELF-PACED LEARNING MODULE: TECHNOLOGY FOR TEACHING AND LEARNING


Prepared by: John Louie S. Marasigan
Republic of the Philippines
Laguna State Polytechnic University
Province of Laguna
ISO 9001:2015 Certified
Level I Institutionally Accredited

Strategy for Solving Related Rates Problems

i. Draw the figure and label the variables and constants. Use t for time . Assume
all variables are differentiable functions of t.

ii. Find an equation relating the quantity with the unknown rate of change to
quantities whose rates of change are known. Do not substitute the known values
of the variables until after taking the derivative.

iii. Differentiate both sides of this equation with respect to time and solve for the
derivative that will give the unknown rate of change.

iv. Substitute into the resulting equation all known values for the variables and
their rates of change. Then solve for the required rate of change.

Example. (A ladder sliding down) The top of a ladder 10 meters long rests on a
vertical wall of a residential building while the bottom rests on a horizontal
ground. If the top slides down at the rate of 20 meters per minute, how fast is the
lower end moves along the ground when the lower end is 8 meters from the wall.

Solution: We draw the figure and label the desired quantities

Source Note: Comandante, Felipe L. (2005) Differential Calculus made Easy (Metric Edition) National Bookstore

Let t = time elapsed since the ladder


started to slide down

y = distance from the ground


to the top of the ladder

x = distance from the bottom


of the ladder to the wall

LSPU SELF-PACED LEARNING MODULE: TECHNOLOGY FOR TEACHING AND LEARNING


Prepared by: John Louie S. Marasigan
Republic of the Philippines
Laguna State Polytechnic University
Province of Laguna
ISO 9001:2015 Certified
Level I Institutionally Accredited
𝑑𝑥
We want to find 𝑑𝑡 when x = 8 meters, now using the Pythagorean Theorem we
can describe the relationship between x and y. Hence,

(10)2 = 𝑥 2 + 𝑦 2

Since x and y are function of time t, we can differentiate both sides of the
equation with respect to t and obtain

𝑑𝑥 𝑑𝑦 𝑑𝑥 𝑦 𝑑𝑦
0 = 2x 𝑑𝑡 + 2y 𝑑𝑡 ⟹ =- 𝑥 ∙ 𝑑𝑡
𝑑𝑡

𝑑𝑥 𝑑𝑦
We can find 𝑑𝑡 , since x = 8 meters, y =6 meters and = -20 meters per minute.
𝑑𝑡
Hence,
𝑑𝑥 6 𝑚 𝑑𝑥 𝑚
=- 8 ∙ (−20 𝑚𝑖𝑛) ⟹ = 15𝑚𝑖𝑛
𝑑𝑡 𝑑𝑡

Therefore, the ladder moves along the ground at the rate of 15 meters per
minute.

Example: (A moving shadow) A man 167.68-centimeter-tall is walking directly


away form a lamp post fronting a building at the rate of 91.46 centimeters per
second. If the lamp is 8 meters above the ground, find
a.) the rate at which his shadow is lengthening and
b.) the rate at which the tip of his shadow is moving

Solution: We draw the figure and label the desired quantities.

Source Note: Comandante, Felipe L. (2005) Differential Calculus made Easy (Metric Edition) National Bookstore

Let s = length of man’s shadow


x = distance of the man from the lamp post
𝑑𝑠
= rate at which the man’s shadow is lengthening
𝑑𝑡
𝑑𝑤
= rate at which the tip of the man’s shadow is moving
𝑑𝑡
𝑑𝑥
= rate at which the man is moving away from the lamp post
𝑑𝑡

LSPU SELF-PACED LEARNING MODULE: TECHNOLOGY FOR TEACHING AND LEARNING


Prepared by: John Louie S. Marasigan
Republic of the Philippines
Laguna State Polytechnic University
Province of Laguna
ISO 9001:2015 Certified
Level I Institutionally Accredited
a.) By similar triangles, we can find the rate at which the man’s shadow is
lengthening
𝑠 𝑥 +𝑠
= ⟹ s = 0.27x
167.68 800
𝑑x
Differentiating (1) with respect to t, we obtain (notice that = 91.46 cps)
𝑑𝑡

𝑑𝑠 𝑑𝑥 𝑑𝑠
= (0.27) ∙ ⟹ = 24.69 𝑐𝑝𝑠
𝑑𝑡 𝑑𝑡 𝑑𝑡

b.) Let w = x + s. Notice that the tip of the man’s shadow is moving in a line , so
we differentiate w with respect to t and get

𝑑𝑤 𝑑𝑥 𝑑𝑠
w=x+s ⟹ 𝑑𝑡
= 𝑑𝑡
+ 𝑑𝑡

𝑑𝑠 𝑑𝑥
Substituting 𝑑𝑡 = 24.69 cps and 𝑑𝑡 = 91.46 cps into the preceding equation, we
get the rate at which the tip of the man’s shadow is lengthening.

𝑑𝑤
= (24.69) + (91.46) = 116.15 cps
𝑑𝑡

Maxima and Minima Involving Algebraic Functions


In the previous chapter, we have seen the geometrical interpretation of
derivative of a function as the slope of a curve at the point which is just equal to
the slope tangent line. We have also discussed the derivative as a rate of change.

In this section, we use the derivative to find the extreme values of a function the
relative maximum and relative minimum values, which enable us to graph
polynomial and rational functions effectively.

First, we introduce some definition and theorems to classify the relative maxima
and relative minima of functions.

The Relative Maximum Value

The function f(x) is said to have a relative maximum value at x0 if there exists
an open interval containing x0, on which f(x) is defined such that f(x0) ≥ f(x) for
all x in this interval

The Relative Minimum Value


The function f(x) is said to have a relative minimum value at x0 if there exists
an open interval containing x0, on which f(x) is defined such that f(x0) ≤f(x) for
all x in this interval

LSPU SELF-PACED LEARNING MODULE: TECHNOLOGY FOR TEACHING AND LEARNING


Prepared by: John Louie S. Marasigan
Republic of the Philippines
Laguna State Polytechnic University
Province of Laguna
ISO 9001:2015 Certified
Level I Institutionally Accredited

Relative Extrema
Source Note: Comandante, Felipe L. (2005) Differential Calculus made Easy (Metric Edition) National Bookstore

The Relative Extremum

A function f(x) is said to have a relative extremum at x0 if it has either a relative


maximum or a relative minimum at x0 (See Figure Above)

Remarks

The existence of a relative maximum (or minimum) of a function at a point does


not imply that it has largest (or smallest) value at that point.

Theorem

If f(x) exists for all values of x in the open interval (a,b) and if f(x) has a relative
extremum at x0, where a< x0 < b, and if f’(x0) exists , then f’(x0) = 0 of f(0) is not
differentiable at x0.

The geometrical interpretation of this theorem is that if f(x) has a relative


extremum at x0 and if f’(x0) exists, then the graph of f(x) must have a horizontal
tangent line at the point where x = x0.

The Critical Number

The critical number of a function f(x) is any value of x in the domain of f(x) at
which f ‘(x)= 0 or at which f(x) is not differentiable. The relative extrema of a
function, if any, occur at the critical numbers.

The Extreme Value Theorem

If a function f is continuous on a closed interval [a,b] the f has an absolute


maximum and an absolute minimum value at least once in [a,b]

LSPU SELF-PACED LEARNING MODULE: TECHNOLOGY FOR TEACHING AND LEARNING


Prepared by: John Louie S. Marasigan
Republic of the Philippines
Laguna State Polytechnic University
Province of Laguna
ISO 9001:2015 Certified
Level I Institutionally Accredited
The Absolute Maximum Value

f(x0) is said to be absolute maximum value of the function f(x) if x0 is in the


domain of f(x) and f(x0) ≥ f(x) for all values of x in the domain of f(x). (See Figure
below)

The Absolute Minimum Value

f(x0) is said to be absolute minimum value of the function f(x) if x0 is in the


domain of f(x) and f(x0)≤ f(x) for all values of x in the domain of f(x). (See Figure
below)

Strategy for Obtaining the Absolute Extrema of a Function

i. Find all the critical number x0 of f(x) on (a, b)


ii. Calculate f(x0) for each of the critical number x0.
iii. Find the value of f(a) and f(b)
iv. The largest of the values form steps i to iii is the absolute maximum
value and the smallest of the values is the absolute minimum value.

Absolute Relative Extrema


Source Note: Comandante, Felipe L. (2005) Differential Calculus made Easy (Metric Edition) National Bookstore

𝑥3
Example 1. If y = f(x) = 4 − 3𝑥, find the absolute maximum or minimum values
of f on the closed interval [-3,3]. Sketch the graph of f .

Solution. Using the given strategy to find the absolute maximum or minimum,
we begin by finding the critical numbers of f. Now, differentiating the given
function with respect to x , we get

𝑥3 𝑑𝑦 3𝑥 2
y= − 3𝑥 ⟹ = −3
4 𝑑𝑥 4

LSPU SELF-PACED LEARNING MODULE: TECHNOLOGY FOR TEACHING AND LEARNING


Prepared by: John Louie S. Marasigan
Republic of the Philippines
Laguna State Polytechnic University
Province of Laguna
ISO 9001:2015 Certified
Level I Institutionally Accredited
𝑑𝑦
Solving 𝑑𝑥 = 0 on x , we have

3𝑥 2
−3=0 ⟹ x = 2, -2
4

Thus, the critical numbers are x1 = 2 and x2 = -2 . Since f is continuous on [-3,3]


it follows form our discussion that the absolute maximum and minimum are
among the values f(-3) , f(-2) , f(2) and f(3). Calculating these values, we obtain

(−3)3
f(-3) = − 3(−3) = 2.25 ⟹ ( -3 , 2.25)
4
(−2)3
f(-2) = − 3(−2) =4 ⟹ ( -2 , 4)
4
(2)3
f(2) = − 3(2) = -4 ⟹ (2 , -4)
4
(3)3
f(3) = − 3(3) = - 2.25 ⟹ ( 3 , - 2.25)
4

𝑥3
Therefore, the absolute maximum point of f(x) = 4 − 3𝑥 on [-3, 3] is the point (2,
-4) and it is a relative minimum point and the point (-2,4) is absolute maximum
point and it is also relative maximum point (See Figure Below)

Source Note: Comandante, Felipe L. (2005) Differential Calculus made Easy (Metric Edition) National Bookstore

Applied Optimization Problems

To optimize something means to make it as useful or effective as possible. We


will use the Extreme Value Theorem to solve applied optimization problems.
Such type of problems can be solved using the techniques for finding the
absolute maximum or minimum of a function

In this section, we will deal with problems that are not originally stated with
mathematical equations; hence additional formulations of variables are required
in the solution process.

LSPU SELF-PACED LEARNING MODULE: TECHNOLOGY FOR TEACHING AND LEARNING


Prepared by: John Louie S. Marasigan
Republic of the Philippines
Laguna State Polytechnic University
Province of Laguna
ISO 9001:2015 Certified
Level I Institutionally Accredited
Strategy for Solving Optimization Problems

i. Read the problem carefully. Assign variables to all quantities involved


in the problem. When feasible, make a sketch and label it
appropriately.
ii. Translate all relationships into mathematical equations, then identify
the variables to be maximized or minimized and express it as a
function of one variable
iii. Determine the values of the variable for which the stated problem
make sense
iv. Apply the Extreme Value Theorem to Determine the Desired
maximum or minimum.

Example: Find the dimensions of the rectangular lot that can enclosed using
1,000m of fencing material and whose area is as large as possible

Solution: We draw the figure and label the desired quantities

Rectangular Lot
Source Note: Comandante, Felipe L. (2005) Differential Calculus made Easy (Metric Edition) National Bookstore

Let x = length of the rectangular lot


y = width of the rectangular lot
A = area of the rectangular lot
P = perimeter of the rectangular lot

Our goal is to find the dimensions of the rectangular lot whose area is as large as
possible with a given perimeter. The area of the rectangular lot is given by

𝐴 = 𝑥𝑦

Since the perimeter of the lot is 1,000 meters, we relate the variables x and y by
the equation

𝑃 = 2𝑥 + 2𝑦 ⇒ 2𝑥 + 2𝑦 = 1000

𝑦 = 500 − 𝑥

LSPU SELF-PACED LEARNING MODULE: TECHNOLOGY FOR TEACHING AND LEARNING


Prepared by: John Louie S. Marasigan
Republic of the Philippines
Laguna State Polytechnic University
Province of Laguna
ISO 9001:2015 Certified
Level I Institutionally Accredited
By substituting ,
𝐴 = 𝑥 (500 − 𝑥 ) ⟹ 𝐴 = 500𝑥 − 𝑥 2

Differentiating with respect to x then solving for x , we obtain

𝑑𝐴
= 500 − 2𝑥 ⟹ 500 – 2x = 0 ⟹ x = 250 m
𝑑𝑥

By substitution again we get

𝑦 = 500 − 250 ⟹ 𝑦 = 250 𝑚

Since the two perpendicular sides are equal, we say that the rectangle formed is
a square with sides equal to 250 m. We can solve the maximum area using the
formula, 𝐴 = 𝑥𝑦. Hence,

𝑨 = 𝟐𝟓𝟎(𝟐𝟓𝟎) = 𝟔𝟐, 𝟓𝟎𝟎 𝒔𝒒. 𝒎𝒆𝒕𝒆𝒓𝒔

Example : (Finding the maximum area) A first class radio-telephone operator


wishes to cut the wire 100 meters long into two pieces. One of the pieces will be
bent into the shape of a square and the other into the shape of a circle. Where
should he cut the wire so that the sum of the areas of the square and the circle is
maximized?

Solution: We draw the figure and label the desired quantities

Source Note: Comandante, Felipe L. (2005) Differential Calculus made Easy (Metric Edition) National Bookstore

Let x = one of the cut pieces of the wire


100 – x = length of the other pieces

and let the piece of length x be bent to form a square of side s so that

1
4𝑠 = 𝑥 ⇒ 𝑠= 𝑥
4

LSPU SELF-PACED LEARNING MODULE: TECHNOLOGY FOR TEACHING AND LEARNING


Prepared by: John Louie S. Marasigan
Republic of the Philippines
Laguna State Polytechnic University
Province of Laguna
ISO 9001:2015 Certified
Level I Institutionally Accredited
If the remaining piece is bent into a circle of radius r, then

100 − 𝑥
2𝜋𝑟 = 100 − 𝑥 ⇒ 𝑟=
2𝜋

The area of a square is given by 𝐴𝑠 = 𝑠 2 . Substituting into on the above


equation, we get

1 1 2
𝐴𝑠 = ( 𝑥)2 ⇒ 𝐴𝑠 = 𝑥
4 16

The area of a circle is defined by 𝐴𝑐 = 𝜋𝑟 2 . Substituting into the above equation,


yields

𝑥 2 (100 − 𝑥)2
𝐴𝑡 = 𝐴𝑠 + 𝐴𝑐 ⇒ 𝐴𝑡 = +
16 4𝜋

We can find the critical numbers by differentiating the above equation with
respect to x , we have

𝑑𝐴𝑡 𝑥 100 − 𝑥
= −
𝑑𝑥 8 2𝜋
𝑑𝐴𝑡
Solving = 0 for x in the above equation
𝑑𝑥

𝑥 100 − 𝑥
− =0 ⇒ 𝑥 = 56 𝑚
8 2𝜋
Hence, the length of the wire to be bent into the shape of a square is 56m.
Now, we can find the length of the wire to be bent into the shape of a circle in the
following manner,
100 − 𝑥 = 100 − 56 = 44 𝑚

To find the maximum area ,

(56)2 (100−56)2
𝐴𝑡 = + ⇒ 𝐴𝑡 = 350.1 𝑠𝑞. 𝑚
16 4𝜋

LSPU SELF-PACED LEARNING MODULE: TECHNOLOGY FOR TEACHING AND LEARNING


Prepared by: John Louie S. Marasigan
Republic of the Philippines
Laguna State Polytechnic University
Province of Laguna
ISO 9001:2015 Certified
Level I Institutionally Accredited
First Derivative Test

Without calculus we do not know the exact behavior of the curve between the
points we plot. Even if we plot hundreds of points and connect them with line
segments, the maximum and minimum values of the curve could not be
determined accurately.

In this section, we will use derivatives to determine the behavior of a continuous


function on a certain interval whether it is increasing or decreasing and to find
the relative extrema of a function on that interval.

The Increasing Function

A function 𝑓(𝑥) defined on an interval is said to be increasing on that interval


of and only if
𝑓(𝑥1 ) < 𝑓(𝑥2 ) whenever 𝑥1 < 𝑥2

where 𝑥1 and 𝑥2 are any numbers in the interval

𝑓 is increasing on [a, b]
Source Note: Comandante, Felipe L. (2005) Differential Calculus made Easy (Metric Edition) National Bookstore

The Decreasing Function

A function 𝑓(𝑥) defined on an interval is said to be decreasing on that interval


of and only if
𝑓(𝑥1 ) > 𝑓(𝑥2 ) whenever 𝑥1 < 𝑥2

where 𝑥1 and 𝑥2 are any numbers in the interval

𝑓 is decreasing on [a, b]
Source Note: Comandante, Felipe L. (2005) Differential Calculus made Easy (Metric Edition) National Bookstore

LSPU SELF-PACED LEARNING MODULE: TECHNOLOGY FOR TEACHING AND LEARNING


Prepared by: John Louie S. Marasigan
Republic of the Philippines
Laguna State Polytechnic University
Province of Laguna
ISO 9001:2015 Certified
Level I Institutionally Accredited
Theorem

Let 𝑓(𝑥) be a function that is continuous on a closed interval [𝑎, 𝑏] and


differentiable on the open interval (a,b)
i. If 𝑓 ′ (𝑥) > 0 for every x in (a,b) then 𝑓(𝑥) is increasing on [a,b]
ii. If 𝑓 ′ (𝑥) < 0 for every x in (a, b) then 𝑓(𝑥) is decreasing on [a,b]

Example: Let 𝑦 = 𝑓 (𝑥 ) = 𝑥 3 − 3𝑥 2 − 9𝑥 , find the intervals on which 𝑓 is


increasing and the intervals on which 𝑓 is decreasing. Sketch the graph of 𝑓.

Solution: Differentiating 𝑦 with respect to 𝑥, then solving for 𝑥, yields


𝑑𝑦
= 3𝑥 2 − 6𝑥 − 9 ⟹ 3(𝑥 + 1)(𝑥 − 3) = 0
𝑑𝑥

𝑥1 = −1 and 𝑥2 = 3

To find the x-intercepts in the given function, we set 𝑦 = 0 then solve for 𝑥.
Hence,

0 = 𝑥(𝑥 2 − 3𝑥 − 9) ⟹ 𝑥 = 0 , 4.9 𝑎𝑛𝑑 − 1.85


From the previous theorem, it is sufficient to find the intervals in which 𝑓 ′ (𝑥) >
0 and those values in which 𝑓 ′ (𝑥 ) < 0. See table below

Graph of 𝑓 (𝑥 ) = 𝑥 3 − 3𝑥 2 − 9𝑥
Source Note: Comandante, Felipe L. (2005) Differential Calculus made Easy (Metric Edition) National Bookstore

LSPU SELF-PACED LEARNING MODULE: TECHNOLOGY FOR TEACHING AND LEARNING


Prepared by: John Louie S. Marasigan
Republic of the Philippines
Laguna State Polytechnic University
Province of Laguna
ISO 9001:2015 Certified
Level I Institutionally Accredited
The First Derivative Test

Suppose 𝑓(𝑥 ) is continuous at a critical number 𝑥0 and differentiable on an open


interval (a, b) containing 𝑥0 , except possibly at 𝑥0 itself.

i. If 𝑓 ′ (𝑥) is positive for all 𝑥 (∀𝑥) to the left of 𝑥0 and negative ∀𝑥 to the
right of 𝑥0 , then 𝑓(𝑥0 ) is a relative maximum value of 𝑓 (𝑥 )

ii. If 𝑓 ′ (𝑥) is negative for all 𝑥 (∀𝑥) to the left of 𝑥0 and positive ∀𝑥 to the
right of 𝑥0 , then 𝑓(𝑥0 ) is a relative minimum value of 𝑓 (𝑥 )

iii. If 𝑓 ′ (𝑥) > 0 or if 𝑓 ′ (𝑥 ) < 0 for every 𝑥 in the interval except at 𝑥 = 𝑥0


then 𝑓(𝑥0 ) is no relative extremum of 𝑓 (𝑥 ). This means the no
relative extremum occurs at those critical points where the sign of the
first derivative does not change moving in the positive x-direction.

Relative Extrema
Source Note: Comandante, Felipe L. (2005) Differential Calculus made Easy (Metric Edition) National Bookstore

Example: Find the relative extrema and sketch the graph of


1
𝑦 = 2 (𝑥 − 1)2 (4 − 𝑥)

Solution: Differentiating 𝑦 with respect to 𝑥 , then solving for the critical


numbers, yields
1
𝑦 = (𝑥 − 1)2 (4 − 𝑥)
2
𝑑𝑦 1
= − (𝑥 − 1)2 + (4 − 𝑥)(𝑥 − 1)
𝑑𝑥 2
1
(𝑥 − 1)( − 2 (𝑥 − 1)2 + (4 − 𝑥 )) = 0

Solving the first factor for 𝑥 , we get

𝑥1 − 1 = 0 ⟹ 𝑥1 = 1

LSPU SELF-PACED LEARNING MODULE: TECHNOLOGY FOR TEACHING AND LEARNING


Prepared by: John Louie S. Marasigan
Republic of the Philippines
Laguna State Polytechnic University
Province of Laguna
ISO 9001:2015 Certified
Level I Institutionally Accredited
Solving the second first factor for 𝑥 , we have

1
− (𝑥 2 − 1) + ( 4 − 𝑥 2 ) = 0 ⟹ 𝑥2 = 3
2

Therefore, the critical numbers are 𝑥1 = 1 and 𝑥2 = 3. See figure and table
below

1
Graph of 𝑦 = (𝑥 − 1)2 (4 − 𝑥)
2
Source Note: Comandante, Felipe L. (2005) Differential Calculus made Easy (Metric Edition) National Bookstore

Concavity and the Second Derivative Test

We have observed the behavior of the function 𝑓(𝑥 ) as it increases or


decreases on a certain interval using the first derivative test.
In this section, we will extend this idea by locating the interval in which
the graph of 𝑓 (𝑥 ) is concave upward or concave downward.

LSPU SELF-PACED LEARNING MODULE: TECHNOLOGY FOR TEACHING AND LEARNING


Prepared by: John Louie S. Marasigan
Republic of the Philippines
Laguna State Polytechnic University
Province of Laguna
ISO 9001:2015 Certified
Level I Institutionally Accredited
The Concavity
Let 𝑓 (𝑥 ) be a function that is differentiable on an open interval 𝐼. The graph of
𝑓(𝑥 ) is defined as

i. Concave upward on 𝐼 if 𝑓 ′ (𝑥 ) is increasing on 𝐼, or the graph of 𝑓(𝑥 )


is above the tangent line on every point on that interval
ii. Concave downward on 𝐼 if 𝑓 ′ (𝑥 ) is decreasing on 𝐼, or the graph of
𝑓(𝑥 ) is below the tangent line on every point on that interval

Graphical Representation of Concavity


Source Note: Comandante, Felipe L. (2005) Differential Calculus made Easy (Metric Edition) National Bookstore

The Test for Concavity

Let 𝑓 (𝑥 ) be a function whose second derivative exists on an open interval 𝐼.


i. If 𝑓 ′ ′(𝑥) > 0 for all 𝑥 in 𝐼, then the graph of 𝑓(𝑥) is concave upward on
𝐼.
ii. If 𝑓 ′ ′(𝑥) < 0 for all 𝑥 in 𝐼, then the graph of 𝑓(𝑥) is concave downward
on 𝐼.

The Point of Inflection

If the graph of a continuous function 𝑓(𝑥) possesses a tangent lime at a point


where its concavity changes from upward to downward (or vice versa), then the
point is called a point of inflection. See figure below

Types of Points of Inflection


Source Note: Comandante, Felipe L. (2005) Differential Calculus made Easy (Metric Edition) National Bookstore
Remarks

At a point of inflection, the graph crosses its tangent line. It occurs where the
concavity of the graph changes. Thus, to locate the possible points of inflection,
we need only to determine the value of x for which 𝑓 ′ ′(𝑥) = 0 or for which 𝑓 ′ ′(𝑥)
does not exist.

LSPU SELF-PACED LEARNING MODULE: TECHNOLOGY FOR TEACHING AND LEARNING


Prepared by: John Louie S. Marasigan
Republic of the Philippines
Laguna State Polytechnic University
Province of Laguna
ISO 9001:2015 Certified
Level I Institutionally Accredited
The Second Derivative Test

Let 𝑓 ′ ′(𝑥 ) exists on some open interval containing 𝑥0 and 𝑓 ′ (𝑥0 ) = 0


i. If 𝑓 ′′ (𝑥0 ) > 0 , then 𝑓(𝑥0 ) is relative minimum value.
ii. If 𝑓 ′′ (𝑥0 ) < 0 , then 𝑓(𝑥0 ) is relative maximum value.
iii. If 𝑓 ′′ (𝑥0 ) = 0 , then the test fails

Remarks
We can use this test if the second derivative exists. This test is based on the fact
that if 𝑓(𝑥0 ) is a relative maximum value, then its graph is concave downward
in some interval containing 𝑥0 . Similarly, if 𝑓(𝑥0 ) is a relative minimum value,
then the graph of 𝑓 is concave upward in some interval containing 𝑥0 .

But if 𝑓 ′′ (𝑥0 ) > 0, then the Second Derivative Test fails. In such cases, we can use
the First Derivative Test.

Notice that if 𝑥0 is a root of odd order(i.e. simple, triple, etc.) of the equation
𝑓 ′ (𝑥0 ) = 0 then 𝑥0 is a relative maximum or minimum; if 𝑥0 is a root of even
order, 𝑥0 is a point of inflection with horizontal tangent.

Example : Use the Second Derivative Test to find the relative extrema , discuss
concavity, points of inflection and sketch the graph of 𝑦 = 𝑥 3 − 3𝑥 + 3, 𝑥 ∈
[−2, 2].

Solution. We begin by finding the critical numbers of the given function.


Differentiating 𝑦 with respect to 𝑥, we get

𝑑𝑦
𝑦 = 𝑥 3 − 3𝑥 + 3 ⟹ = 3𝑥 2 − 3
𝑑𝑥
𝑑𝑦
Solving 𝑑𝑥 = 0 for 𝑥, we obtain the critical numbers: x =-1 and x =1

Using the Second Derivative Test, we get

𝑑2𝑦
= 6𝑥
𝑑𝑥 2

when x = -1 , 𝑓(−1) = (−1)3 − 3(−1) + 3 = −1 + 3 + 3 = 5 ⟹ (−1, 5)

𝑑2 𝑦
= 6(-1) = -6 < 0
𝑑𝑥 2

Thus, the point (-1,5) is a relative maximum point (See Figure below)

LSPU SELF-PACED LEARNING MODULE: TECHNOLOGY FOR TEACHING AND LEARNING


Prepared by: John Louie S. Marasigan
Republic of the Philippines
Laguna State Polytechnic University
Province of Laguna
ISO 9001:2015 Certified
Level I Institutionally Accredited

when x = 1, 𝑓(1) = (1)3 – 3(1) + 3 = 1 – 3 + 3 = 1 ⟹ (1,1)

𝑑2 𝑦
= 6(1) = 6 > 0
𝑑𝑥 2

Thus, the point (1 , 1 ) is a relative minimum point.

𝑑2 𝑦
To find the possible points of inflection, we solve 𝑑𝑥 2 = 0 for 𝑥. Hence,

6𝑥 = 0 ⟹ 𝑥=0

when 𝑥 = 0 , 𝑓 (0) = (0)3 − 3(0) + 3 = 3 ⟹ (0,3)

Thus (0, 3) is a point of inflection.

We summarize the result in the table below

Graph of 𝑦 = 𝑥 3 − 3𝑥 + 3
Source Note: Comandante, Felipe L. (2005) Differential Calculus made Easy (Metric Edition) National Bookstore

LSPU SELF-PACED LEARNING MODULE: TECHNOLOGY FOR TEACHING AND LEARNING


Prepared by: John Louie S. Marasigan
Republic of the Philippines
Laguna State Polytechnic University
Province of Laguna
ISO 9001:2015 Certified
Level I Institutionally Accredited
Applied Maxima and Minima
(Comandante, 2005)

In this section, we will show how the methods developed in the previous section
can be used to solve more applied optimization problems.

Strategy for Solving Applied Maxima and Minima


i. Set up an equation whose maximum or minimum value is required.
Make the necessary figures or illustrations to make the solution
clearer and easier
ii. If the resulting equation involves more than one variable, express
these relations in terms of one variable using the sufficient conditions
given in the problem
iii. Apply the first and second derivative tests.

Example: A fire has occurred on a fire storey townhouse. Find the minimum
length of a ladder that will reach from the ground to the fifth floor of the
townhouse. The ladder should rest on a fence 𝑥 meters high is 𝑏 meters from the
townhouse to rescue the victims trapped on the fifth floor.

Solution:
We draw the figure and label the desired quantities (See Figure Below)

Ladder Reaching the Fifth Floor of a Townhouse Over a Fence


Source Note: Comandante, Felipe L. (2005) Differential Calculus made Easy (Metric Edition) National Bookstore

Let 𝐿 = length of the ladder


𝑥 = height of the fence
𝑦 = height of the fifth floor of the townhouse from the ground
𝑏 = distance from the fence to the townhouse
𝑎 = distance from the fence to the lower edge of the ladder

LSPU SELF-PACED LEARNING MODULE: TECHNOLOGY FOR TEACHING AND LEARNING


Prepared by: John Louie S. Marasigan
Republic of the Philippines
Laguna State Polytechnic University
Province of Laguna
ISO 9001:2015 Certified
Level I Institutionally Accredited
From similar triangles, we get

a a+b 𝑥(𝑎+𝑏)
= ⟹ 𝑦=
x y 𝑎

The length of the ladder is

𝐿2 = (𝑎 + 𝑏)2 + 𝑦 2

By substitution, we obtain

𝑥(𝑎+𝑏) 2 (𝑎+𝑏)√𝑎 2+ 𝑥 2
𝐿2 = (𝑎 + 𝑏)2 + ( ) ⟹ 𝐿=
𝑎 𝑎

Differentiating 𝐿 with respect to 𝑎(𝑏 𝑎𝑛𝑑 𝑥 𝑎𝑟𝑒 𝑓𝑖𝑥𝑒𝑑), then solving for 𝑎, we get

1 −1
𝑎((𝑎 + 𝑏) ∙ (2 (𝑎2 + 𝑥 2 ) 2 (2𝑎)) + √𝑎2 + 𝑥 2 (1) − (𝑎 + 𝑏)√𝑎 2 + 𝑥 2 (1))
𝑑𝐿
=
𝑑𝑎 𝑎2

𝑑𝐿 𝑎3 − 𝑏𝑥 2
=
𝑑𝑎 𝑎3 √𝑎2 + 𝑥 2

𝑎3 − 𝑏𝑥 2 3
=0 ⟹ 𝑎 = √𝑏𝑥 2
𝑎 3 √𝑎2 + 𝑥 2

Hence, we have a unique critical number 𝑎. By substitution , we get

3 3
( √𝑏𝑥2 +𝑏)√( √𝑏𝑥2 )2+ 𝑥 2 2 2 3
𝐿= 3 ⇒ 𝐿 = (𝑥 3 + 𝑏 3 )2
√𝑏𝑥2

LSPU SELF-PACED LEARNING MODULE: TECHNOLOGY FOR TEACHING AND LEARNING


Prepared by: John Louie S. Marasigan
Republic of the Philippines
Laguna State Polytechnic University
Province of Laguna
ISO 9001:2015 Certified
Level I Institutionally Accredited
Example: An open box having a square base is to be constructed from 𝑘 square
meters of materials. What dimensions should be used for the box to obtain a
maximum volume?

Solution We draw the figure and label the desired quantities (See Figure Below)

Open Box with Square Box


Source Note: Comandante, Felipe L. (2005) Differential Calculus made Easy (Metric Edition) National Bookstore

Let 𝑥 = side of the box


ℎ = height of the box
𝑉 = volume of the box
𝑆 = surface area of the box

The volume of the box (𝑉 = 𝑙𝑒𝑛𝑔𝑡ℎ ∙ 𝑤𝑖𝑑𝑡ℎ ∙ 𝑡ℎ𝑖𝑐𝑘𝑛𝑒𝑠𝑠) is

𝑉 = 𝑥 2ℎ
The surface area 𝑆 of the box is

𝑆 = (𝑎𝑟𝑒𝑎 𝑜𝑓 𝑡ℎ𝑒 𝑏𝑎𝑠𝑒) + (𝑎𝑟𝑒𝑎 𝑜𝑓 𝑡ℎ𝑒 𝑓𝑜𝑢𝑟 𝑠𝑖𝑑𝑒𝑠)

𝑆 = 𝑥 2 + 4𝑥ℎ ⟹ 𝑥 2 + 4𝑥ℎ = 𝑘

Now since the volume 𝑉 of the box is to be maximized, it is helpful to express 𝑉


As a function of one variable, say 𝑥. To do this, we solve (2) for ℎ in terms of 𝑥
and 𝑘 to get
𝑘 − 𝑥2
ℎ=
4𝑥

By substitution, it yields

𝑘 − 𝑥2 1
𝑉 = 𝑥2 ⟹ 𝑉= 𝑥 (𝑘 − 𝑥 2 )
4𝑥 4

LSPU SELF-PACED LEARNING MODULE: TECHNOLOGY FOR TEACHING AND LEARNING


Prepared by: John Louie S. Marasigan
Republic of the Philippines
Laguna State Polytechnic University
Province of Laguna
ISO 9001:2015 Certified
Level I Institutionally Accredited
Differentiating, with respect to 𝑥 and obtain the critical number, we have

1 𝑘
(𝑘 − 3𝑥 2 ) = 0 ⟹ 3𝑥 2 = 𝑘 ⟹ 𝑥= √
4 3
We only consider the positive value of 𝑥. Differentiating, we get

𝑑2𝑉 3 3
= ( −2 ) ( ) 𝑥 = − 𝑥
𝑑𝑥 2 4 2

Substituting,
𝑑2 𝑉 3 𝑘
= − 2 (√ 3 ) < 0
𝑑𝑥 2

𝑘
Therefore, the volume of the box is maximum when 𝑥 = √3

Differential

𝑑𝑦
In the previous sections, we only consider the symbol 𝑑𝑥 or 𝑓′(𝑥) as the first
derivative of the function with respect to 𝑥. However, it is useful to be able to
𝑑𝑦
think of as the quotient of two symbols 𝑑𝑦 and 𝑑𝑥. The following definition
𝑑𝑥
𝑑𝑦
will enable us to think of 𝑑𝑥 as either the derivative of the function with respect
to x it is the ratio of 𝑑𝑦 and 𝑑𝑥.

The Differential of 𝒙.

If 𝑦 = 𝑓 (𝑥 ) is a differentiable function of 𝑥, then the differential of 𝑥, 𝑑𝑥 is an


increment of 𝑥 , that is, 𝑑𝑥 = ∆𝑥. The differential of 𝑦 is 𝑑𝑦 = 𝑓 ′ (𝑥 )𝑑𝑥. Notice that
in the equation 𝑑𝑦 = 𝑓 ′ (𝑥 )𝑑𝑥 if both sides are divided by 𝑑𝑥 then

𝑑𝑦
= 𝑓′(𝑥) , 𝑑𝑥 ≠ 0
𝑑𝑥

𝑑𝑦
Thus, 𝑑𝑥 can be thought of as the symbol for the first derivative of 𝑓′(𝑥) or as the
quotient of differentials. Although it is true that 𝑑𝑥 = ∆𝑥 , in general, 𝑑𝑦 ≠ ∆𝑦

LSPU SELF-PACED LEARNING MODULE: TECHNOLOGY FOR TEACHING AND LEARNING


Prepared by: John Louie S. Marasigan
Republic of the Philippines
Laguna State Polytechnic University
Province of Laguna
ISO 9001:2015 Certified
Level I Institutionally Accredited

Differential
Source Note: Comandante, Felipe L. (2005) Differential Calculus made Easy (Metric Edition) National Bookstore

The tangent to the curve at point A is the line through points A and D. The
derivative evaluated at point A gives the slope of the curve which is just the slope
of the tangent line. But the ratio of the lengths of the line segment 𝐷𝐶̅̅̅̅ to 𝐴𝐶
̅̅̅̅ is
also the slope of the tangent line (See Figure above). Thus,

̅̅̅̅
𝐷𝐶
𝑓 ′ (𝑥 ) = ̅̅̅̅
𝐴𝐶

But the length of ̅̅̅̅


𝐴𝐶 is dx, therefore 𝑑𝑦 must be equal to the length of ̅̅̅̅
𝐷𝐶 , hence

̅̅̅̅
𝐷𝐶 ̅̅̅̅
𝐷𝐶
𝑓 ′ (𝑥 ) = =𝑑𝑥 ⟹ ̅̅̅̅
𝐷𝐶 = 𝑓 ′ (𝑥 )𝑑𝑥 = 𝑑𝑦
̅̅̅̅
𝐴𝐶

For any change, ∆𝑥 in 𝑥 we obtain

∆𝑦 = 𝑓 (𝑥 + ∆𝑥 ) − 𝑓(𝑥)

This shown in Figure 4.32 where the difference between ∆𝑦 and 𝑑𝑦 is given by
̅̅̅̅
the line segment 𝐵𝐷

Remarks
Although 𝑑𝑦 ≠ ∆𝑦, it is a good estimate of ∆𝑦 when 𝑑𝑥 is small. If 𝑑𝑥 were made
very small, then the difference between 𝑑𝑦 and ∆𝑦 would also become very
small.

Example We use formula 𝑑𝑦 = 𝑓 ′ (𝑥 )𝑑𝑥. Differentiating 𝑦 with respect to 𝑥 , we


get
𝑑𝑦 𝑑𝑦
= 𝑓 ′ ( 𝑥 ) = 4(3)𝑥 3 + 4 ⟹ = 𝑓 ′ (𝑥 ) = 12𝑥 3 + 4
𝑑𝑥 𝑑𝑥

So that the differential 𝑑𝑦 can be written as 𝑑𝑦 = 𝑓 ′ (𝑥 )𝑑𝑥. Hence,

𝑑𝑦 = (12𝑥 3 + 4)𝑑𝑥.

LSPU SELF-PACED LEARNING MODULE: TECHNOLOGY FOR TEACHING AND LEARNING


Prepared by: John Louie S. Marasigan
Republic of the Philippines
Laguna State Polytechnic University
Province of Laguna
ISO 9001:2015 Certified
Level I Institutionally Accredited
The Error Propagation

i. ∆𝑦 = 𝑓 (𝑥 + ∆𝑥 ) − 𝑓(𝑥) - represent a propagated error

ii. 𝑥 - represents the measured value of a variable

iii. 𝑥 + ∆𝑥 - represents the exact value.

iv. ∆𝑥 - represent the error in measurement

∆𝑦
v. – represent the relative error
𝑦

Example . The measurement of the radius of a circle is found to be 3 meters with


a possible error of 0.00025 meter. Use differentials to approximate the possible
error , relative error and the percentage error in computing the area of the circle.

Solution: Let 𝑟 = 3 m-radius of the circle, and 𝑑𝑟 = 0.00025 possible error

The area of a circle is 𝐴 = 𝜋𝑟 2 ⟹ 𝐴 = 9𝜋

To approximate the possible propagated error in computing the area of the


circle, we computed it as follows . Differentiating 𝐴 with respect to 𝑟 we get the
propagated error. Hence,

𝑑𝐴
= 2𝜋𝑟 ⟹ ∆𝐴 = 𝑑𝐴 = 2𝜋𝑟𝑑𝑟 ⟹ 𝑑𝐴 = 2(3)(0.00025)𝜋
𝑑𝑟

𝑑𝐴 = 0.0015𝑑𝜋 𝑚2

The relative error in computing the area of a circle can be computed as follows,

𝑑𝐴 2𝑑𝑟 𝑑𝐴 2(0.00025) 𝑑𝐴
= ⟹ = ⟹ = 0.000167
𝐴 𝑟 𝐴 3 𝐴

The corresponding percentage error is found to be

𝑑𝐴 𝑑𝐴
(100%) = 0.000167 (100%) ⟹ (100%) = 0.0167%
𝐴 𝐴

LSPU SELF-PACED LEARNING MODULE: TECHNOLOGY FOR TEACHING AND LEARNING


Prepared by: John Louie S. Marasigan
Republic of the Philippines
Laguna State Polytechnic University
Province of Laguna
ISO 9001:2015 Certified
Level I Institutionally Accredited

Performance Tasks

Problem Set:

1. (Water level rising) Water runs into a conical tank at the rate of 8 cubic meters per hour. IF the
height of the cone is 10 meters and the diameter of its opening is 12 meters, how fast is the
water level rising when the water is 3 meters deep?

2. Find the absolute maximum and minimum of y = x3 – 3x2 + 2, [-1,3]. Sketch the graph

3. A farmer finds that if she plants 50 trees per acre, each tree will yield 65 bushels of fruit. She
estimates that for each additional tree planted per acre, the yield of each tree will decrease by 3
bushels. How many trees should she plant per acre to maximize her harvest?

4. The function 𝑓 (𝑥) = (2𝑥 − 1)𝑒 −6𝑥 has one critical number. Find it and then determine if there is
a relative max or min at that x-value. Determine the relative max or min.

5. Determine where the function is concave up and concave down. State any points of inflection
𝑓 (𝑥 ) = 𝑥 4 − 4𝑥 3 + 3

6. When a cubical metal box of edge is 𝑥 is heated, each edge increases by 0.01 per degree increase
in temperature. Show that the surface area of the cube increases by 0.02 per degree and that the
volume increases by 0.03 per degree.

LSPU SELF-PACED LEARNING MODULE: TECHNOLOGY FOR TEACHING AND LEARNING


Prepared by: John Louie S. Marasigan
Republic of the Philippines
Laguna State Polytechnic University
Province of Laguna
ISO 9001:2015 Certified
Level I Institutionally Accredited

Understanding Directed Assess

Rubric

Source: https://studylib.net/doc/6971944/mathematics-problem-solving-rubric

LSPU SELF-PACED LEARNING MODULE: TECHNOLOGY FOR TEACHING AND LEARNING


Prepared by: John Louie S. Marasigan
Republic of the Philippines
Laguna State Polytechnic University
Province of Laguna
ISO 9001:2015 Certified
Level I Institutionally Accredited

Learning Resources

Louis Leithold (1996) The Calculus 7 Haper Collins College Publishers

Comandante, Felipe L. (2005) Differential Calculus made Easy (Metric Edition) National
Bookstore

Cabero, J. et al (2008 ) Solved Problems in DIFFERENTIAL CALCULUS National Bookstore

Maurice D. Weir and Joel Hass. (2010) Calculus 7th Edition Pearson

S. Thompson and M. Gardener (1998) CALCULUS MADE EASY – A VERY SIMPLE


INTRODUCTION TO DIFFERENTIAL AND INTEGRAL CALCULUS St. Martin’s Press

F. Ayres and E. Medelson (2013) Schaum’s Outline of Calculus, 6th Edition Mc Graw Hill

Mark Ryan (2016) Calculus for Dummies (2nd Edition) John Wiley & Sons

Lumen Learning (n.d.) Retrieved August 20, 2020, from


https://courses.lumenlearning.com/suny-openstax-calculus1/

Mathispower4u by James Sousa -Creative Commons Attribution-ShareAlike 3.0 Unported


License (n.d.) Retrieved August 20, 2020, from http://www.mathispower4u.com/

Penn State Department of Mathematics (2020) Retrieved August 21, 2020, from
https://math.psu.edu/undergraduate/courses/math110

Paul’s Online Notes (2003-2020) Retrieved August 15, 2020, from


https://tutorial.math.lamar.edu/

The University of British Columbia mathematics Department Retrieved August 20, 2020,
from http://www.math.ubc.ca/~maclean/math101/MATH-101-learning-
outcomes.pdf

Note: The materials provided to you are for educational purposes only. Please do not share
these materials with anyone else. No part of it can be reproduced for commercial
distribution through manual and electronic means without the knowledge of the authors
or copyright owners.

LSPU SELF-PACED LEARNING MODULE: TECHNOLOGY FOR TEACHING AND LEARNING


Prepared by: John Louie S. Marasigan

You might also like